You are on page 1of 78

Civil Procedure Outline

Jurisdiction over the Parties: In Personam Jurisdiction in State Courts

Chapter 1: A Survey of the Civil Action


A. The Concern and Character of Civil Procedure 1. Two types of Law - Substantive: substantive rights and duties that regulate the everyday relationships among individuals and between individuals and institutions *Also defines the standard of liability in a case - Procedural: procedures used by the courts to resolve the disputes that are brought before them 2. Civil versus Criminal - Civil: initiated and litigated by private parties attempting to vindicate their legal rights vis--vis other private parties - Criminal: instituted and prosecuted by the government in an effort to punish those individuals whose conduct has violated the communitys moral judgments 3. Adversary System - Almost total responsibility placed on the parties to the controversy for beginning suit, for shaping the issues, and for producing evidence * Civil law countries employ the inquisitorial system in which the court conducts an active and independent inquiry into the merits of each case B. An Outline of the Procedure in a Civil Action 1. To consider when suing - Is the grievance one for which the law furnishes relief? - What is the probability of winning a lawsuit? - Is what is won worth the time, the effort, and the expense? * Consider settlement, arbitration, rest 2. Steps in the Civil Procedure - Selecting a Proper Court * Must have jurisdiction over the subject matter and jurisdiction over the person * State Court organization: original jurisdiction in which cases are brought and tried; appellate jurisdiction only to review the decisions of lower courts; intermediate courts of appellate jurisdiction as well * Federal Court organization: United States District Courts; 13 United States Courts of Appeal; The Supreme Court of the United States

~Jurisdiction when matter involves federal law, diversity of citizenship, amount in controversy exceeds $75,000 * Must also have venue - Commencing the Action * Service of Process: includes summons * Personal service, substituted service (registered mail, through and agent), publication -Pleading and Parties * Complaint: written statement that contains claim against defendant * Objective of pleadings: identify and separate the legal and factual contentions involved so that the case may be disposed of; establish in advance what a party intends to prove at trial so the opponent can prepare; give each party a general notice of opponents contentions - The Response * Motion to dismiss: challenge court jurisdiction, service of process, or venue; also for failure to state a claim or cause of action (demurrer) ~ Granted when: injury is one for which the law furnishes no redress; plaintiff failed to include an allegation on necessary part of case; complaint is so general or so confused that it does not give adequate notice of what plaintiffs claim is * Defendant answer: admit or deny the factual allegations; counterclaim; implead separate party - Obtaining Information Prior to Trial * Discovery through: depositions, written interrogatories, production of documents, requests for admissions, physical examinations - Summary Judgments * Demonstrating that the crucial issue will have to be resolved in the movers favor at trial, because the opposing party will be unable to produce any admissible evidence in support of her position on the issue - Setting the Case for Trial - The Jury and its Selection * Assured by 7th Amendment - The Trial * Hearsay evidence: repeats what someone else has said for the purpose of proving the truth of what was said * Directed verdict: no prima facie case (plaintiff has not introduced enough evidence to permit the jury to find in her favor - Submitting the Case to the Jury * Three types of verdicts: general, general with interrogatories, special

- Post-Trial Motions * Judgment notwithstanding the verdict (judgment n.o.v.): same as directed verdict * New trial - The Judgment and its Enforcement - Appeal * Either affirm, reverse, or modify trial court judgment * Remand to trial court - The Conclusiveness of Judgments * Res judicata: a thing decided C. A Note on Motion Practice D. A Note on Remedies 1. Relief a Court may Award: Declarative, specific, compensatory - Declarative: defining the rights and duties of the parties in a particular legal context - Specific: order directing conduct - Compensatory: defendant pays plaintiff a certain sum of money Definition -Civil procedure is the study of lawsuits and the rules that govern the operation of the system and conduct -Very statutory: dealing with written rules -Civil procedure is the official dispute resolution for states Example S [PA] v B [NJ] in PA ct S seeking $100,000 for oral agreement to buy land (Blackacre) that was never fulfilled B alleges 1. no contract made: purely fact, must have trier of fact or juries- substantive legal rules between buying and selling 2. even if he did it is unenforceable under the statue of frauds: certain contracts are legally unenforceable unless they are written and signed: pure question of law for court- substantive 3. court lacks jurisdiction over his person (in personam jurisdiction): mixed question of law (can the court exercise jurisdiction) and fact (under certain circumstances) -B lives in N.J., Blackacre is in NJ, B says he has never even been to PA Pennoyer v. Neff P. 13 M 95 U.S. (5 Otto) 714 (1877) Facts: In 1866, J. H. Mitchell brought action against Marcus Neff to recover unpaid legal fees in an Oregon court. Neff was not a resident of Oregon. Service of process was made by publication in an Oregon newspaper. Neff did

not appear and judgment was entered upon his default. Neffs land in Oregon was publicly sold to Sylvester Pennoyer at a sheriffs sale. Procedure: Neff brought an action of ejectment to recover possession of a piece of land in an Oregon federal court. Found in favor of Neff. Pennoyer appealed to U.S. Supreme Court which affirmed. Issue: Is a monetary judgment, rendered in one State against the resident of another as a result of a simple contract, valid without service of process in person and the appearance of the defendant? Held: No. Due process requires appearance or personal service before the defendant can be personally bound by any judgment. Analysis: The court relied on two principles to decide this case. First, every State possesses exclusive jurisdiction and sovereignty over person and property within its territory. It thus has the power to determine the civil status and capacities of its inhabitants. Second, no State can exercise direct jurisdiction and authority over person or property without its territory. Thus, as the issue concerned a property that resided within Oregon state, the court had jurisdiction to demand payment from the out of state resident to a resident within the State. A state may satisfy the claims of its citizens. Therefore, there was no issue in the court deciding for Mitchell in the initial case. However, the court realized that without personal service of summons, the defendant would likely never see the publication, and as a result fraud and oppression would become commonplace. Publication is only acceptable for proceedings in rem, or against a thing. Pennoyer v Neff Mitchell v Neff -Mitchell filed suit to recover legal fees -Neff not served with process, notice of the suit was published in an Oregon newspaper, judgment against him due to his default, writ of execution delivered to the sheriff after which he conducted a sheriffs sale. Pennoyer purchased the land. -Neff did not appear: means he did not defend the lawsuit Neff v Pennoyer -Action of ejectment not seeking money, but possession of property -Neff prevailed, Supreme Court affirms. -Diversity of citizenship, thus the original case was held in the U.S. District Court for the District of Oregon (about 25% of cases heard in federal court are there because of diversity, but they do not have to be there. It is not exclusive to the federal courts) -Neff contends that the judgment in the state court was void because he was not personally served with process and he was not there to defend the action. The default action was void as the court would not have jurisdiction. Therefore, Pennoyer would have gotten a void title to the property.

-Two established principles: *Every State possesses exclusive jurisdiction and sovereignty over persons and property within its territory *No State can exercise direct jurisdiction and authority over persons or property without its territory- fundamental issue in the case. Neff was outside the territory, Oregon could not exercise jurisdiction over him, unless Mitchell could catch him in the territory and serve him with process. -Neff owned property in Oregon, so Mitchell could have proceeded in rem and attached the property to the original lawsuit (a direct proceeding against the property) not an in personam proceeding against Neff. The property is subject to Oregon jurisdiction So the State, through its tribunals, may subject property situated within its limits owned by non-residents to the payment of the demand of its own citizens against them -Wheatons Treatise on International Law: applied to states -If Mitchell had sued and won damages in Oregon and brought the damages to Neffs state to seek enforcement, then it would be a different story. -To proceed in rem, the plaintiff must essentially sue a piece of land instead of the property. Attachment (tangible as in land) or garnishment (intangible such as bank account) or seizure: it must be brought under the control of the court. The defendant would then know of the action as a result of the seizure of the property. -How would Mitchell seize the land? Court would issue a writ of attachment. Sheriff would place notice on the property. Still, no notice is given. Plaintiff must record the notice of attachment at the recorders office. Very little likelihood that notice is given. -If Mitchell had proceeded in rem and all the other series of events were the same, this would not have been an issue. -If the court allows plaintiff to run ad as the first step, the defendant would never be able to defend. They would be the constant instruments of fraud and oppression. This was really the only policy reason the court uses. -Suppose Mitchell filed suit in Oregon, and knows Neff lives in San Francisco and sends the Oregon sheriff there to serve process. Neff ignores this and the default judgment is entered against him. Pennoyer buys the property, and Neff brings action. Does he have a claim? Yes, Oregon cannot serve process to Neff out of state and would thus still not have jurisdiction. It does not matter how much notice is given. -Court was not entirely concerned with notice. The court was more concerned with the fact that Neff was not served in Oregon. -Suppose Mitchell did exactly the same thing and takes $40,000 judgment to Neff in PA against property in PA. Does the court have to give full faith and credit to the judgment?

* Would sue Neff on the judgment * Only if it is a jurisdictionally valid judgment * Mitchell would be bound by the Pennsylvania judgment E. Pennoyer v Neff Notes 1. Defendants must respond to suits within a stated period to avoid a default judgment 2. In personam: jurisdiction based on power over the defendants person. Operates to impose a personal obligation upon the defendant, such as an obligation to pay money 3. In rem: founded only upon the presence of property within the state - Resulting judgment is limited to the property which supports jurisdiction and does not impose a personal liability on the property owner, since he is not before the court - Binding on all persons with respect to their interest in the property involved, regardless of whether or not they were parties to or had actual notice of the action 4. Quasi-in-rem: determines the interests in the property involved only or those persons who are parties to the proceeding and has no effect on the interest of the persons not made a party. - Plaintiff seeks to secure a preexisting claim in a subject property and extinguish or establish the nonexistence of similar interests of particular persons. Only relates to the two people involved. - Plaintiff seeks to apply what he concedes to be the property of the defendant to the satisfaction of a claim against him. Serves as a substitute for or a supplement to in personam jurisdiction *Attachment, garnishment, seizure of the property to commence an action 5. Exceptions to personal service - Defendant assents to the jurisdiction by voluntary appearance or given in advance - State has jurisdiction to determine the status of one of its own residents even in an action against a nonresident defendant 6. Jurisdictional Rules - Transient jurisdiction: personal service of process within the states territory establishes a state courts personal jurisdiction over the defendant, however unrelated to the plaintiffs cause of action and however brief or even fortuitous the defendants presence in the state may have been *Not used for corporations - Residence of the defendant in the forum state is an independent basis for personal jurisdiction *Corporations subject

- Nonresident can consent to the personal jurisdiction of a states court * voluntary appearance, consent in advance - Presence doctrine: A foreign corporation is amenable to process to enforce a personal liability in the absence of consent, only if it is doing business within the state in such manner and to such extent as to warrant the inference that it is present there. - Exercise jurisdiction over a nonresident in an action brought by a resident to determine the residents status Full Faith and Credit Clause (in supplement) as it applies to judgments - Full faith and credit shall be given in each State to the public acts, records, and judicial proceedings of every other State. *Judgments rendered in other states *See analysis for Pennoyer v. Neff -Suppose Pennoyer v Neff occurred today. Mitchell files suit in Oregon court knowing he cant get jurisdiction. Neff has no property in Oregon. Neff takes vacation in Hawaii, plane makes emergency landing in Oregon and serves him with process. Neff ignores and default judgment is entered. Mitchell brings judgment to PA. Do they have to uphold the full faith and credit clause? *Yes, due to Grace v. McArthur and transient jurisdiction. -Full Faith and Credit had no application to the case because everything happened in Oregon *Spoke about it because the tests are the same under due process and full faith and credit *To exercise in personam jurisdiction: must have service of process in person within the state or voluntary appearance (this has changed) *Exceptions to exercise jurisdiction Background of law between Pennoyer and International shoe -A person is always subject to the jurisdiction in the state in which he is domiciled *What does domicile mean though? *Established in Milliken v Meyer -The defendant can consent to the jurisdiction of a foreign court: voluntarily appears *Consent in advance of the litigation: done often in contracts *State may be able to require consent if the defendant wants to carry on certain activities in the state *Useful when dealing with corporations: corporations have no physical dimensions ~States have the power to exclude corporations from doing business ~They can set conditions for doing business in the state

~Must appoint an in-state agent to receive service of process ~What if they ignore this? Any foreign corporation that in fact did business within the state and did not register would be deemed to have appointed an in-state agent to receive service of process- implied consent -Advent of the automobile: non-resident motorist statutes or long-arm statutes *If a non resident wished to operate a motor vehicle on the highways it would be required to appoint an agent for service of process within the state (Kane v New Jersey) *People did not know of the new statues: if they operated a motor vehicle on the highways in the state they were deemed to have appointed in state agents for service of process *Hess v Pawloski: implied consent. Hess was served on the registrar of motor vehicles. He appeared specially to defend the lawsuit and to challenge the jurisdiction ~Consent is going to be implied regardless of whether or not the defendant knew about it or consent or not ~Consent really has nothing to do with it. -Courts that did purely interstate business could not be excluded -Courts must rely on the manner and extent of the corporations business to determine if it had impliedly consented International Shoe Co. v. Washington P. 33 M 326 U.S. 310, 66 S. Ct. 154, 94 L.Ed. 95 (1945) Facts: International Shoe Co. is a Delaware corporation, with its principal place of business in St. Louis, Missouri. It has never maintained any place of business or stock of merchandise in Washington. It makes no contracts for the sale or purchase of goods in Washington and makes no deliveries of goods there. Between 1937 and 1970 International Shoe Co. employed 11 to 13 salesmen who resided in Washington. They exhibited samples and solicited orders in Washington. They were not authorized to make sales or collections. Orders were sent, filled, and paid for by International Shoe in St. Louis. The salesmen rented hotel rooms or store fronts to display their goods. Procedure: Washington brought action to recover unpaid employment compensation taxes. International Shoe appeared specially asserting they were not under Washington court jurisdiction. Washington Supreme Court rejected this. U.S. Supreme Court affirmed. Issue: Did International Shoe Co. have certain minimum contacts with the state of Washington as to consider it present in the state and thus satisfy due process to assert jurisdiction? Held: Yes. The activities carried on by International Shoe Co. were neither irregular nor casual. Analysis: Relied on the assertion that due process requires only that in order to subject a defendant to a judgment in personam, if he is not present in the

territory, he must have certain minimum contacts with it such that the maintenance of the suit does not offend traditional notions of fair play and substantial justice. The court says that presence is assumed when the corporations activities are continuous and systematic and give rise to the suit. The court recognizes that causal presence is not enough to subject it to suit on actions unrelated to the activities in the state. This would be an unreasonable burden on the corporation. Due process depends on the quality and nature of the activity. If a corporation has the privilege to conduct activities in the state and thus enjoy the benefit and protection of the laws, it has certain obligations. F. International Shoe Co. v. Washington notes 1. Special appearance: permits defendant to appear in court for the sole purpose of challenging the courts jurisdiction without thereby submitting to that jurisdiction. International Shoe Co. v Washington -If this were decided simply based on the law decided in McKibbin (see p.30 M) it would still have determined that jurisdiction was valid: corporate presence -Activities were systematic and continuous; claim by the plaintiff arose out of the activities of the defendant in the state: presence have never been doubted -Solicitation plus some additional activities there were necessary to subject the corporation to the jurisdiction: mere solicitation of orders would not do it -Associated with minimum contacts test so that it is fair, just, and reasonable to hold the corporation subject to jurisdiction *Relationship between forum and corporation *Relationship between forum and plaintiffs claim -Opposite end: no contacts, no in personam jurisdiction- rarely found. Plaintiffs dont sue in states in which the defendant has absolutely zero contacts whatsoever -Applicable to persons as well - Contacts that are systematic and continuous, contacts that are related to the cause of action Note 3 Page 38 - No systematic and continuous activity in the forum state. Only a single isolated action. - Plaintiffs claim arose out of defendants activity - Single or isolated act coupled with a related activity: maybe jurisdiction. I.e. Kane v New Jersey and Hess v. Pawloski *Exactly the situation in McGee Note 4

- The activities of the company are systematic and continuous. Unrelated cause. *General jurisdiction *How systematic and continuous do the activities need to be?

Chapter 2: Jurisdiction over the parties or their Property


A. Due Process and Long-Arm Statues McGee v. International Life Insurance Co. P. 89 C 355 U.S. 220, 222-224, 78 S. Ct. 199, 200-201, 2 L.Ed.2d 223, 225-226 (1957) Facts: McGee was beneficiary of a life insurance policy issued to Lowell Franklin, a resident of California. International Life conducted business through mail with Franklin until his death in 1950. They never had an office or agent in California, nor had they ever solicited or done any insurance business in California other than with Franklin. They refused to pay McGee upon his death, so she sued in a California court. Procedure: McGee recovered judgment in California, tried to enforce in Texas. Texas court refused holding it void on the ground that California could not assume jurisdiction over International Life without service of process in its boundaries. Supreme Court reversed. Issue: Was the exercise of jurisdiction over International Life in California proper? Held: Yes. Analysis: McGee sued basing jurisdiction on the California Unauthorized Insurers Process Act, which subjects foreign corporations to suit on insurance contracts with in-state residents. The court discussed the current state of commerce and the overwhelming amount of business done across state lines. They said the suit was based on a contract which had substantial connections to California. McGee v. International Life Insurance Co. - Nature of the contract was important. It was a continuing relationship. - California Insurance Code Single or isolated contacts Unrelated activity NO NO jurisdiction Single or isolated Related activity McGee Maybe Systematic and continuous Unrelated activity Perkins General Jurisdiction Maybe jurisdiction

No contacts

Systematic and continuous Related Jurisdiction

jurisdictio n

jurisdiction Hanson v. Denckla P. 90 C 357 U.S. 235, 78 S.Ct. 1228, 2 L.Ed.2d 1283 (1958)

Facts: Dora Donner of Pennsylvania established a trust in Delaware with a Delaware bank as trustee. She then moved to Florida and upon her death her estate was left to her two daughters. She also designated her two grandchildren as beneficiaries of a significant portion of the trusts assets. After her death, the two daughters brought suit in Florida claiming the appointment of the granddaughters as beneficiaries was ineffective. The defendants argued the Florida court could not assert jurisdiction over the Delaware trustee. Procedure: Action in Florida, found it had jurisdiction, concluded trust was invalid. Action commenced in Delaware to determine who was entitled to share the trusts assets. Delaware held trust and exercise of appointment was valid. Supreme Court granted certiorari for both cases. Issue: Could Florida assert jurisdiction over a Delaware trustee? Held: No. The trustees contacts with Florida were less than minimal. Florida did not obtain personal jurisdiction over the trustee, so Delaware was justified in refusing full faith and credit. Analysis: The court failed to find certain minimum contacts required to assert personal jurisdiction. The trust company has no offices in Florida and transacts no business there. None of the trust assets have ever been held or administered in Florida and there was no solicitation of business. The court also said that the cause of action did not arise out of activities in the state. Justice Black dissented as he considered the fact that the appointment of the granddaughters as beneficiaries occurred in Florida and the primary beneficiaries lived in the state. He believed the Florida court had jurisdiction unless it would impose a huge burden and would offend traditional notions of fair play and substantial justice. B. General Jurisdiction and State Long-Arm Statues Perkins v. Benguet Consolidated Mining Co. P. 125 C 357 U.S. 235, 78 S.Ct. 1228, 2 L.Ed.2d 1283 (1958) Facts: Philippine corporation was sued by a nonresident of Ohio in an Ohio state court. The plaintiff claimed $68,000 due in dividends as a stockholder and $2,500,000 in damages due to the companys failure to issue her certificates for 120,000 shares of its stock. Procedure: Motion to quash the service of summons was granted by the Ohio state courts.

Issue: Does the due process clause of the Fourteenth Amendment prohibit Ohio from exercising jurisdiction? Held: No. The corporations activities were both continuous and substantial in Ohio. Analysis: The court looked to see if the companys activities in the state were continuous and systematic. Those activities are enough to make it fair and reasonable to subject that corporation to proceedings in personam in that state, at least insofar as the proceedings in personam seek to enforce causes of action relating to those very activities or to other activities of the corporation within the state. However, in this case, the court considered the fact that the suit did not arise out of the corporations activities in the state. Instead, they consider the substantiality and nature of the activities in the state. Perkins v Benguet - Perfect case to demonstrate general jurisdiction *How systematic and continuous do those activities have to be? ~Significantly more contacts 1. Fisher Governor Co. v. Superior Court: must look beyond sales activities in the state to determine if jurisdiction is valid 2. Frummer v. Hilton Hotels: jurisdiction upheld on the basis of a separate corporation than the one being sued, because there was interlocking ownership 3. Ratliff v. Cooper Laboratories, Inc.: South Carolina could not assert jurisdiction over two drug companies as their contacts were insufficient to satisfy due process. Plaintiffs chose the state for its long statue of limitations. The drugs were manufactured and consumed outside of the state and the cause of action was unrelated to the activities in the state. Helicopteros Nacionales de Colombia, S.A. v. Hall P. 128 C Supreme Court of the United States, 1984 466 U.S. 408, 104 S.Ct. 1868, 80 L.Ed.2d 404 Facts: Helicopter owned by Helicol crashed in Peru. Four U.S. citizens died. They worked for Consorcio which used helicopter transport to transport personnel, materials, and equipment. The CEO previously had met Consorcio in Houston, Texas to discuss business matters, yet the agreement was signed in Peru. Helicol purchased approximately 80% of its fleet from Bell Helicopter Company in Texas. They sent pilots to Texas for training and sent management and maintenance personnel to Bell Helicopter to receive plant familiarization. Procedure: Wrongful death action initiated in Texas against Consorcio, Bell Helicopter, and Helicol. Helicol filed special appearances, moved to dismiss the action for lack of in personam jurisdiction. Motion denied. Judgment entered against Helicol. Judgment reversed on appeal for lack of jurisdiction. Supreme

Court of Texas reversed intermediate court decision. Writ of certiorari granted. Reversed. Issue: Did the Supreme Court of Texas correctly rule that the contacts of a foreign corporation with the State of Texas were sufficient to allow a Texas state court to assert jurisdiction over the corporation in a cause of action not arising out of or related to the corporations activities with the state? Held: No. Helicols contacts with the State of Texas were insufficient to satisfy the requirements of the Due Process Clause. Analysis: The court relied on the principal that even if the cause of action does not arise out of activities in the state, that jurisdiction can be exercised if there are certain minimum contacts with the state. They do not believe that purchases and related training trips are sufficient contact with the state required to assert in personam jurisdiction. They relied on the decision in Rosenberg Bros. & Co. v Curtis Brown Co. which makes clear that purchases and related trips standing alone, are not a sufficient basis for a States assertion of jurisdiction. Justice Brennan dissented, however he believed the courts conclusion was not implausible or unexpected. His main concern is the implication that may be drawn from the decision. He believed that the contacts were not only sufficient but that they were related to the cause of action in the suit. He claimed that the negotiations that took place in Texas lead to the contract in which Helicol agreed to provide the transportation services that were being used at the time of the crash. The helicopter was also purchased in Texas and the pilot trained there as well. Brennan believes that a courts specific jurisdiction should be applicable whenever the cause of action arises out of or relates to the contacts between the defendant and the forum. Helicopteros - Rejects claim of general jurisdiction - Substantial purchases made from Texas: over $4 million over eight years - Always had one or more of its employees in Texas over the eight year period - Contract for services was negotiated in Texas, executed in Peru - Continuing activity were the major purchases made *Just the purchase of goods, according to the court, is not enough *Court seemed to look at this acts in accumulation - The court ONLY considered general jurisdiction, not specific -Theory of pilot negligence: Brennan in dissent notes that the helicopter was purchased in Texas, the pilot was trained in Texas, Helicol in Texas negotiated this specific contract to render the services in Peru -But forargument. But for the negotiation, the helicopter in question would not be in Peru *Distinction between arising out of and related to But For Causation

-But for A, B would not have occurred. -Substantive relevance to a claim: essential element of the plaintiffs claim *Proximate cause approach: more direct -Sliding scale approach: greater degree of contact between defendant and forum, less connection required -Realistic approach: fact oriented on a case by case basis C. Specific Jurisdiction and State Long-Arm Laws 1. Long-arm or single-act statues: seek to provide personal jurisdiction over nonresidents who cannot be found and served in the forum - predicate jurisdiction over nonresidents upon the defendants general activity in the state, or the commission of any one of a series of enumerated acts within the jurisdiction, or, in some cases, the commission of certain act outside the jurisdiction causing consequences within it. -Apply only to suits brought in the courts of the state in which the jurisdictional act occurs or in the federal courts sitting in that state Gray v. American Radiator & Standard Sanitary Corporation Supreme Court of Illinois, 1961 22 Ill.2d 432, 176 N.E.2d 761 Facts: Phyllis Gray was injured when her water heater exploded and she was injured. She filed suit against the Titan Valve Manufacturing Company alleging that they negligently constructed the safety valve and the injuries were suffered as a proximate result thereof. The summons was served on Titans registered agent in Cleveland, Ohio. Titan has no agent physically in Illinois, does no business there, and that it sells the completed valves to American Radiator & Standard Sanitary Corporation which is outside of Illinois. Procedure: Action for damages in circuit court of Cook county. Dismissed. Titan filed motion to quash on the ground that it did not commit a tortuous act in Illinois. Granted. Gray appeal to Supreme Court of Illinois since there is a question of constitution. Reversed and remanded. Issue: Was a tortuous act committed in Illinois and does the statue violate due process of law? Held: Yes; No. A tortuous act was committed in Illinois and the defendants association with this State is sufficient to support the exercise of jurisdiction. Analysis: The court looked at section 17 (1)(b) which says that a nonresident who, either in person or through an agent, commits a tortious act within this State submits to jurisdiction. The court did not dispute that a tortious act was committed. Instead, they were unsure whether the act was committed in Illinois. The court says the wrong occurred from acts performed at the manufacturer. Only the consequences were in Illinois. The court did not believe the negligence in manufacturing could be separated from the resulting negligence, and thus the act occurred in Illinois.

The defendants claim that the injury is their only contact with the state, thus exceeding the limits of due process. The court then analyzed cases that dealt with minimum contacts required to assert jurisdiction. The court says that the volume of business done is not the only way to determine minimum contacts. Gray v. American Radiator - Titans only place of business was in Ohio - American Radiator incorporated the value into their product: Pennsylvania company - The radiator moved in the course of commerce to Illinois -If a statutory question is affirmed, then there can be a due process question - States do not have an inherent authority to exercise long-arm jurisdiction. Only done if authorized by a long-arm statue -Titans act was manufacturing the defective valve. The consequences were realized in Illinois, the act cannot be separated from the result of the act. The place where the last even takes place which is necessary to impose liability. *Exercise jurisdiction over anyone who commits a tortuous act within the state. Focus is on where the act occurs, instead of where the tort occurs. *To be tortious and act must cause injury. It was not a tortious act until the plaintiff was injured. -Nelson v. Miller: exercise jurisdiction to the fullest extent permissible by the constitution -Court read the statue as the New York statue -Sold with the expectation that it would wind up somewhere else Supplement long arm statues -Laundry list of circumstances in which the court can exercise jurisdiction extraterritorial -New York: exercise jurisdiction over anyone who commits a tort someplace else that causes injury within the state World-Wide Volkswagen Corp. v. Woodson P.94 Supreme Court of the United States, 1980 444 U.S. 286, 100 S.Ct. 559, 62 L.Ed.2d 490 Facts: Harry and Kay Robinson purchased a new Audi from Seaway Volkswagen, Inc. in Massena, N.Y. in 1976. While they were driving to their new home in Arizona, as they passed through the State of Oklahoma, another car struck their Audi in the rear causing a fire which severely burned Kay and her two children. Suit alleged that the injuries resulted from defective design and placement of the Audis gas tank and fuel system. Defendants included were the manufacturer, Audi NSU Auto Union Aktiengesellschaft, its importer

Volkswagen of America, its regional distributor World-Wide Volkswagen Corporation, and its retail dealer Seaway. Both corporations do no business in Oklahoma, ship or sell no products in the state, does not have an agent to receive process there, and has no advertisements there Procedure: Products liability action in District Court for Creek County, Okla. Writ of prohibition by petitioners due to lack of minimum contacts. Writ denied. Issue: Can an Oklahoma court exercise in personam jurisdiction over a nonresident automobile retailer and its wholesale distributor when its only connection with Oklahoma is the fact that an automobile sold in New York to New York residents became involved in an accident in Oklahoma? Held: No.. The petitioners have no contacts, ties, or relations. Analysis: After reviewing the principals that were established in International Shoe and McGee, the court was unable to find minimum contacts with Oklahoma. It was argued that because a vehicle is mobile, it should have been foreseeable that the car could cross into Oklahoma. However, the court says that foreseeability alone is not enough. The implications would be too steep if foreseeability alone was enough. More importantly the court says the defendants conduct and connection with the forum state should be that which he should reasonably anticipate being haled into court there. Justice Brennan dissented. He believed the court should have given more weight to the strength of the forum States interest in the case and that they didnt consider the inconvenience or lack thereof to the defendant. He believed that the connection between the State and the litigation was strong. Not only did the accident occur in Oklahoma, but the plaintiffs were hospitalized there and essential witnesses and evidence were there. He asserted that the seller purposefully injected the goods into the stream of commerce and that those goods predictably are used in the forum State. World-Wide Volkswagen Corp. v. Woodson -Would consider the decision in Grey wrong. Due process reasoning no longer valid. -Because a vehicle is mobile, it is foreseeable that the car would cross into Oklahoma. Foreseeability alone is not enough, even in suits for claims arising out of defendants activities. *Hanson v. Denkla: it was foreseeable that she would move to Florida and would seek to exercise appointment there, but that alone was not sufficient -Foreseeability that the defendants contacts and conducts are such that he can reasonably anticipate being hailed into court there. When is it reasonably foreseeable? *When it is reasonable for the state to do it? *Purposely avails itself of the privilege of conducting activities within the forum State: invokes benefits of forums states laws

2. Insurance Corp. of Ireland v. Compagnie Des Bauxites de Guinee: personal jurisdiction recognizes and protects and individual liberty interest 3. Keeton v. Hustler Magazine, Inc.: Kathy Keeton of New York brought a libel suit against Hustler, an Ohio Corporation, in New Hampshire. Argued jurisdiction for Hustler as it sold 10,000 to 15,000 magazines there. She helped produce a magazine that was circulated there. Supreme Court said they absolutely had jurisdiction . 4. Kulko v Superior Court: suit for a modification of a child support agreement by a California citizen against her ex-husband, a New Yorker. California jurisdiction over defendant father was upheld by State Supreme Court, as he committed a purposeful act outside of the state (purchasing his daughters airline ticket) that caused an effect within the state. U.S. supreme court reversed because they felt the use of the effects test was wrong. - Effects test continues to have viability but only when the defendants conduct both has an effect in the forum state and was directed at the forum state by the defendant - Committed an intentional act - Expressly aimed at the forum state - Causing harm, the brunt of which is suffered and which the defendant knows is likely to be suffered in the forum state Hypothetical B P 41 -In Woodson, the consumer took the product in Illinois. In Titan, they sold the valve for resale, and it ended up in Illinois. -Argue factual difference should make a legal difference -Assume there is a car dealer located in Camden, and every year 40% of its sales are made to Philadelphians who return to Pennsylvania. P purchases a car from D. P lives in Philadelphia and drives the car there, and the steering wheel disengages and he is injured. He wants to bring action against D in Philadelphia, can he? *Did D advertise in Philadelphia? Is it calculated to the forum? Different courts have held differently regarding national advertising -Assume D does no advertising that can be calculated to reach the Philadelphia market. *Much more foreseeable than in Woodson. Continuous and systematic business with Philadelphians Burger King Corp. v. Rudzewicz P. 108 Supreme Court of the United States, 1985 471 U.S. 462, 105 S.Ct. 2174, 85 L.Ed.2d. 528 Facts: Burger King is a Florida Corporation whose principal offices are in Miami. Day to day monitoring of franchisees is conducted through district offices who report to Miami headquarters. John Rudzewics is a Michigan

resident who entered into a twenty-year franchise contract with Burger King to operate a restaurant in Michigan with his partner Brian MacShara. They fell behind on their monthly payments and Burger King brought a diversity action in Federal District Court in Florida for breach of franchise obligations. Rudzewics and MacShara claimed that because they were Michigan residents and because Burger Kings claim did not arise within Florida, that the district court lack jurisdiction. The court relied on Floridas long-arm statute that says Florida courts have jurisdiction over any person who breaches a contract in the state by failing to perform acts that the contract requires to be performed there. Procedure: Action to recover damages and injunctive relief. Court of appeals reversed holding that district court did not have jurisdiction. Supreme Court reversed and remanded. Issue: Does a Florida District court have jurisdiction over Michigan residents who breach a contract with a Florida corporation? Held: Yes Analysis: The court looked to determine if the defendant purposefully established minimum contacts within the forum State. The court said that although Rudzewics and MacShara had virtually no contacts with Florida, that the franchise dispute grew directly out of a contract with had substantial connection with that State. The court said that Rudzewicz established a substantial and continuing relationship with Burger Kings Miami headquarters, received fair notice from the contract documents and the course of dealing that he might be subject to suit in Florida, and has failed to demonstrate how jurisdiction in Florida would be otherwise fundamentally unfair. Justice Stevens and Justice White dissented. They believed that the respondent only did business in Michigan and his principal contacts with Burger King were in Michigan. He agreed with the appellate court in deeming that nothing in the course of negotiations gave Rudzewicz reason to anticipate suit outside of Michigan. Burger King Corp. v. Rudzewicz -Most recent attempt to define what is meant by minimum contacts -Diversity action -How does a Florida long-arm statue operate in federal court? -Two stage inquiry: *Minimum contacts requirement (purposeful availment by the out of state defendant-direct activities to forum state such that he seeks the benefit of the forum States laws) *Fair play and substantial justice (other factors) -Sliding scale: less of minimum contacts require more fair play and substantial justice and visa versa -This is the test that has been used since Burger King was decided: a lot of judgment required on the court making the opinion -The franchisee had a contact with Florida because they entered into a contract with a Florida corporation: contract alone was not enough

-Like in McGee -He relies on the nature of the contract (20 years, highly intertwined relationship), also on the choice of law provision (generally courts will follow it) *There was no choice of law in Hanson v Denkla, the court must make its own choice of law -Provisions can include consent by the defendant to consent to the jurisdiction of a foreign court -After minimum contacts was shown, defendant would have to make a compelling showing that it was unjust and unreasonable to have to litigate the case in Florida 5. Alchemie International, Inc. v. Metal World, Inc: jurisdiction was found based on defendants calls and mail communications to plaintiff as significant contacts Asahi Metal Industry Co. v. Superior Court P. 117 Supreme Court of the United States, 1987. 480 U.S. 102, 107 S.Ct. 1026, 94 L.Ed.2d 92. Facts: On September 23, 1978, Gary Zurcher lost control of his Honda motorcycle on a California highway and collided with a tractor, thereby sustaining serious injuries and resulting in the death of his wife. He filed a products liability suit in the Superior Court in California alleging the accident resulted by a sudden loss of air and an explosion in the rear tire of the motorcycle. The complaint named Cheng Shin Rubber Industrial Co., Ltd. a Taiwanese manufacturer of the tube. They filed a cross complaint seeking indemnification from Asahi Metal Industry Co which manufactured the tubes valve assembly. A manager of Cheng Shin said he fully believe Asahi was aware that valve stem assemblies sold to Cheng Shin would end up in California. Asahi denied they had any knowledge. Procedure: Asahi moved to quash summons. Superior Court denied. Appeals court reversed demanding the Superior Court quash the summons. Supreme Court reversed. Issue: Whether the mere awareness on the part of a foreign defendant that the component it manufactured, sold, and delivered outside the United States would reach the forum state in the stream of commerce constitutes minimum contacts between the defendant and the forum state such that the exercise of jurisdiction does not offend traditional notions of fair play and substantial justice. Held: No. There must be additional conduct which indicates intent or purpose to serve the forum market. Analysis: The Supreme Court said that the substantial connection between the defendant and the forum State necessary for a finding of minimum contacts must come about by an action of the defendant purposefully directed toward the forum state. The mere placement of a product into the stream of

commerce does not suffice. There must be additional conduct that indicates an intent or purpose to serve the forum market. The court did not find that Asahi designed its product in anticipation of sale in California. The court said reasonableness of the exercise of jurisdiction is based on several factors. The burden on the defendant, the interests of the forum state, the plaintiffs interest in obtaining relief, and the interstate interest are all factors. Applying these facts to the Asahi case it is found that the exercise of jurisdiction is unreasonable. Justice Brennan concurred. He did not agree with the conclusion that Asahi did not purposely avail itself of the California market, but he did agree in the unreasonableness of exercising jurisdiction. He believed that the mere possibility that the final product will be marketed in the forum State is enough to exercise jurisdiction. Asahi Metal Industry v. Superior Court -Pure stream of commerce is not sufficient (stream of commerce plus approach) -Concrete evidence that Asahi products did end up in California in substantial numbers -The California court found that Asahi knew that its products were being sold by Cheng Shin in the United States and specifically in California -Grey would not be good law in Justice OConnors opinion -Grey would be good law in Justice Brennans opinion -Justice Stevens said stream of commerce is not valid in this case. He would look at the volume, the value, and the hazardous character of the components. In this case, delivery of over 100,000 units annually suffices. -What is the law with respect to jurisdiction based on a stream of commerce theory: there is not a majority of the court committed to either position. There is no law. *Lower courts would have to decide based on what they think the law is -If Asahi had been a Pennsylvania corporation: the degree of burden on the defendant would be much lower (they dont have to defendant themselves in a foreign nations legal system) 6. Parry v. Ernst Home Center Corp: The court said without a showing of additional conduct they were unable to find that the eventual sale of a product in Utah justifies personal jurisdiction. 7. Calder v. Jones: Jurisdiction was found over a publisher, writer and editor who all resided in Florida because the magazine sold over 600,000 copies in California. -Application of stream of commerce -Court relied on the fact that they were charged with intentional and allegedly tortious actions that were expressly aimed at California,

and they knew the brunt of the injury would be felt by the plaintiff in California -Effects test *Intentional actions *Expressly aimed at the forum state *Causing harm, the brunt of which was suffered- and the defendant knows is likely to be suffered-in the forum state 8. Issues with the Internet and jurisdiction -The internet does not target any particular state -Sliding Scale Test (Zippo Test) *If the defendant enters into contracts with the residents of foreign jurisdiction that involve knowing and repeated transmissions of files over the internet, jurisdiction is proper *A passive website that simply makes information available to any interested is not ground for jurisdiction (similar to a form of advertising) *Websites where users can exchange information are subject to jurisdiction depending on the level of interactivity and commercial nature of the exchange *Interactive, passive, non-interactive 9. Quill Corp. v. North Dakota: If a foreign corporation purposefully avails itself of the benefits of an economic market in the forum State, it may subject itself to the States in personam jurisdiction even if it has no physical presence in the State. -There was no physical presence of Quill in North Dakota Jurisdiction Based on Power over Property A. Pennoyer v Neff: A state may always exercise jurisdiction over a person served with process within its borders, it may also assert in rem jurisdiction over any property within its borders regardless of the domicile or presence of the owner. B. Harris v. Balk: Harris, a resident of N.C. owed $180 to Balk, a resident of N.C. Balk owed Epstein $344. Harris visited Baltimore and Epstein commenced suit against Balk by attaching the debt. Balk sued Harris in N.C. to recover the debt. N.C. courts held that the Maryland judgment was invalid because the debt was in N.C. and not subject to attachment in Maryland. *Supreme Court disagreed. If someone is temporarily in a state and his creditor sues him there to recover debt, then he is liable to process of garnishment, no matter where the debt was located originally. The obligation to pay a debt travels with the debtor. Harris v. Balk -This idea became very difficult when things became intangible

*Eventually assigned situs of intangible property for jurisdictional purposes C. Tyler v. Judges of the Court of Registration: A proceeding in rem may be instituted and carried to judgment without personal service upon claimants within the state, or notice by name to those outside of it, and not encounter any provision of either constitution. D. Pennington v. Fourth National Bank: Indebtness due from a resident to a nonresident is property within the state. The only essentials to the exercise of the states power are presence of the res within its borders, its seizure at the commencement of proceedings, and the opportunity of the owner to be heard. Basic principal -Attach or garnish or seize or sequester the property at the outset and it was in the jurisdiction then it was subject to attachment and the out of state person would be subject to jurisdiction regarding that property Shaffer v. Heitner P. 146 Supreme Court of the United States, 1977. 433 U.S. 186, 97 S.Ct. 2569, 53 L.Ed.2d 683 Facts: Heitner is a nonresident of Delaware and owns one share of stock in the Greyhound Corp. which is incorporated in Delaware and has principal place of business in Arizona. Heitner filed a shareholders derivative suit in Delaware against 28 of the of the companies executives. He sequestered their Delaware property which was authorized by Delaware statue. He alleged the individual defendants violated their duties to Greyhound and thus the corporation was held liable for damages in an antitrust suit and criminal contempt action. Stock was considered to be in Delaware and thus subject to seizure. Procedure: Motion for order of sequestration of the Delaware property. Defendants motion to quash service of process and to vacate the sequestration order. Court of Chancery rejected. State Supreme Court affirmed. Supreme Court reversed. Issue: Whether the standard of fairness and substantial justice set forth in International Shoe should be held to govern actions in rem as well as in personam. Held: Yes. Analysis: In the past the court held that property cannot be subjected to judgment unless reasonable and appropriate efforts have been made to give the property owners actual notice of the action. An adverse judgment in rem directly affects the property owner by divesting him of his rights in the property. The court said the basis for jurisdiction must be sufficient to justify exercising jurisdiction of the interests of persons in a thing. The appellants holdings in Greyhound do not provide contacts with Delaware sufficient to support jurisdiction. Justice Powell concurred.

Justice Stevens concurred. He said that one who purchases stock on the open market can not be expected to know that they are subject to suit in a forum remote from his residence and unrelated to the transaction. Justice Brennan concurred and dissented. He agreed that the minimum contacts analysis is a sensible construct. He believes that a forum has jurisdiction to adjudicate a shareholder derivative action centering on the conduct and policies of the directors and officers of a corporation chartered by that state. Shaffer v. Heitner -Property is taxed in the state in which it is located -Establishing where the stock is, is important so it can be determined where it will be taxed -The stock was in Delaware -Under the statue, the property did not have to be related to the claim. In this case it was not related. The claim arose out of the corporate directors actions in the company and not their stock. *The defendants only contact was that they owned stock in the Delaware corporation, the property was completely unrelated to the plaintiffs claim against the defendant *There were no other contacts -Minimum contacts standard applied -All proceedings are essentially against persons. You cannot sue property. Attaching property is essentially cutting off the persons interest in the property. -The presence of property alone is okay when someone is injured on the property absent of the owner. Also, when claims to the property itself are the source of the underlying controversy. *Both arise out of the property. They are related in a way that Heitners claims were not *They establish minimum contacts and support in personam jurisdiction -There is not much use left for attachment jurisdiction (there is no more in Pennsylvania) - Shareholders derivative action: a suit by a corporate shareholder to enforce a right or claim of the corporation when its management fails to do so. It protects shareholders financial interest in corporations. E. Rhoades v. Wright: Presence of land and the active use of the land by an out of state owner was sufficient contact with Utah to satisfy due process requirements F. Rush v. Savchuk: Insurance obligations cannot be attached to effect quasiin-rem jurisdiction.

G. In Rem Jurisdiction in the Federal Courts 1. Narrowly available. A district court can exercise attachment jurisdiction only if -Personal jurisdiction over the defendant cannot be obtained with reasonable efforts by service of process in any manner authorized by Rule 4 -Such attachment is authorized by the law of the state where the district court sits Feder v. Turkish Airlines -Decided after Shaffer, upheld quasi-in-rem jurisdiction based on attachment of the defendants New York bank account. If they were unable to do this, they would have had to go to Turkey to litigate. *Jurisdiction by necessity Question E -Based on the idea of implied consent Jurisdiction Based Upon Physical Presence Burnham v. Superior Court P. 159 Supreme Court of the United States, 1990. 495 U.S. 604, 110 S.Ct. 2105, 109 L.Ed.2d 631 Facts: Mrs. Burnham brought a suit for divorce in California in January 1988. In late January, Mr. Burnham visited California on business and to see his children. When he returned the children to Mrs. Burnham, he was served with process. Procedure: Motion to quash summons. Superior Court denied. Appeals court denied. Supreme Court affirmed. Issue: Whether the Due Process Clause of the Fourteenth Amendment denies California courts jurisdiction over a nonresident, who was personally served with process while temporarily in that State, in a suit unrelated to his activities in the State. Held: No. Jurisdiction is okay. Analysis: It is a firmly established principle of personal jurisdiction that State courts have jurisdiction over nonresidents who are physically present in the state. The court could not find a single state or federal statue or a single judicial decision that goes against this view. Its validation is its pedigree. Justice White concurred because the notion is so widely accepted that he would not want to strike it down. Justice Brennan concurred. He would undertake an independent inquiry into the fairness of the prevailing in-state service rule. He had purposefully directed his activities to the state and while there he enjoyed the benefit and protection of California law. Justice Stevens concurred as he agreed with all of the justices.

Burnham v. Superior Court -Transient jurisdiction: service of process on defendant while he is physically present in the state is still good law *Even though it is a totally unrelated claim -General jurisdiction -International Shoe suggests that this is okay -Minimum contacts is not a substitute for physical presence -Was Grace v. McArthur (served while in air) correctly decided? *In Burnham, the defendant was purposely in California. In McArthur, he was flying over the state and had no control. -What is the difference between Brennan (contemporary view) and Scalia (historical view): the state can exert jurisdiction because it could have in 1868 (when the fourteenth amendment was adopted. If the writers thought it was okay, then why wouldnt it be?). *Not only have we been using transient jurisdiction since 1868 but we have continued to use it since then -Note E: The exercise of in rem jurisdiction is essentially in personam -Burnham had notice because that is the way we have always done it. It is just tradition. A. Quill Corp. V. North Dakota: A North Dakota state court can assert personal jurisdiction in a suit brought by the State to enforce its use tax against an outof-state mail-order house with neither outlets nor sales representatives in the State. Contacts were sufficient. B. Jurisdiction based on consent 1. Consent by -Expressly agreeing to submit to the court -Performing certain acts that constitute a waiver of objections to the personal jurisdiction -Failing to assert a defense of lack of jurisdiction 2. Insurance Corp. of Ireland v. Compagnie Des Bauxites De Guinee: Mechanical failure forced a halt in production, CBG files claim which insurers refused to pay. Sued in federal court in P.A. Foreign corporations contested jurisdiction, but their failure to supply the requested information as to their contacts with P.A. supports the presumption that the refusal to produce evidence was but an admission of the want of merit in the asserted defense. 3. Most states, a foreign corporation that registers as a condition of doing business in that state consents to suit in that state. 4. M/S Bremen v. Zapata Off-Shore Co.: International contract. Disputes to be litigated before the London Court of Justices. Suit in Florida. Supreme Court said more weight should have been given to the forum clause, unless it is unreasonable.

5. Carnival Cruise Lines, Inc. v. Shute: Consumer contract. Suit filed in Washington claiming negligence. Supreme Court said forum-selection clause should be enforced. It said all disputes would be resolved in the courts of Florida. Jurisdiction Based on Consent -Forum selection clause: historically they have been unenforceable -Typically are upheld unless the defendant can show it would be unfair Jurisdictional Reach of the Federal District Courts Omni Capital Intl. v. Rudolf Wolff & Co. P 69M Facts: Omni employed Wolff as a broker to handle trades on the London Metals Exchange. Wolff is a British corporation with its offices in London. Omni is a New York corporation. Omni was sued because it was alleged they misrepresented its tax benefits and future profits. Omni impleaded Wolff saying its liability, if any, was due to their improper trading activities. Procedure: Moved to dismiss for lack of personal jurisdiction. District Court concluded it could exercise personal jurisdiction. Motion for reconsideration granted. Concluded it lacked personal jurisdiction. Affirmed by 5th circuit. Affirmed by the supreme court. Issue: Is there authorization to serve summons in this litigation? Held: No. Analysis: Before a federal court may exercise personal jurisdiction over a defendant, the procedural requirement of service of summons must be satisfied. There must be more than notice to the defendant and a constitutionally sufficient relationship between the defendant and the forum. There also must be a basis for the defendants amenability to service of summons. The court relied on Rule 4 of the F.R.C.P. The CEA does not authorize service of summons on Wolff and Gourlay, so they looked to the long-arm statue in Louisiana where the district court sits. This only applies to a defendant who regularly does or solicits business or engages in any other persistent course of conduct, or derives substantial revenue from goods used of consumed or services rendered in the state. Neither part of Rule 4(e) was satisfied. The court was not keen to the idea of creating their own rule authorizing service of process in the litigation. The court said the District Court may not exercise jurisdiction without authorization to serve process. Omni v. Wolff -How did Floridas long-arm statue apply in a federal court (Burger King): Federal Rules of Civil Procedure Rule 4(k)(1)(a). If there is no federal statue, then the state law in which the court sits will apply. -Rule 4(k)(1)(a) was Rule E when Omni was decided -Plaintiffs brought action against Omni in the U.S. District Court for the Eastern District of Louisiana. Defendants impleaded the two British third

party defendants (Wolff and Gourlay) (Authorized by Rule 14(a)provides that a defendant can implead non-parties). Liable over-if Omni is liable to the plaintiff, then third party defendants would be liable to Omni. When the third party defendants are brought in, it is no different as if they had been original defendants. The jurisdiction question and rules would be the same. *Omni referred to as the plaintiff -Commodity Exchange Act did not explicitly or otherwise authorize service of process on the British defendants. -They Louisiana long-arm statue did not apply because they did not meet the terms of the statue. It did not provide for extraterritorial jurisdiction on the facts of this case. Omni had conceded that the statue was not fulfilled. -100 mile bulge rule: service of process within 100 miles of the court. Wolff and Gourlay would have fallen within the rule, but they were not within 100 miles. -Omni could go to Great Britain to pursue their suit. They could sue them in a state whose long-arm statue would apply to them, for example New York. If there is no other state, then there is no federal forum open to Omni. -Rule 4(k)(2)(a): For a claim arising under federal law, serving a summonscan exercise jurisdiction if the defendant is NOT subject to jurisdiction in any state courts general jurisdiction. If there is a state forum open, there is also a federal forum open in that state. *Fallback provision for when no state forum is open, and therefore no federal forum is open *Would have to know the relations between Wolff and Gourlay and other states *The purpose of the rule was to overrule Omni. For things arising under Federal Law, there would be at least one federal forum open -Omni does not give a test to determine if the exercise of jurisdiction by a federal court is consistent with the constitution *We now have rule 4K2: action arising out of federal law, consistent with due process, must confer with state statue -The dissenters said they should fashion a rule to exercise jurisdiction. Republic of Panama v. BCCI Holdings S.A. P. 79 M 119 F.3d 935 (11th Cir. 1997) Facts: The complaint charged each of the defendants with participating in a scheme to assist former Panamanian military officer, Manuel Noriega, in illegally diverting Panamanian government funds for his personal use. Complaint filed in the Southern District of Florida. Procedure: District court dismissed claims against First American Bank for lack of personal jurisdiction. Appeal. Reversed.

Issue: Whether the applicable statue potentially confers jurisdiction over the defendant. Whether the exercise of jurisdiction comports with due process. Held: Yes, the statutory basis is satisfied. No, there is no constitutional impediment to jurisdiction. Analysis: Section 1965(d) of the RICO statue provides for service in any judicial district in which the defendant is found. When a federal statue provides for nationwide service of process, it becomes the statutory basis for personal jurisdiction. The court then looked at the decision in FTC v. Jim Walter Corp. where the fifth circuit concluded that once minimum contacts with the U.S. as a whole have been established, considerations of fairness or reasonableness have no further relevance. The court said that the assertion of federal power should not cause courts to abandon completely their role as protectors of individual liberty and fundamental fairness. They said courts should balance the burdens imposed on the individual defendant against the federal interest involved in the litigation. The court said this should only be considered when a defendant has presented a compelling case that would render jurisdiction unreasonable. Courts should examine a defendants aggregate contacts with the nation a as a whole rather than his contacts with the forum state. Republic of Panama -No problem with the statutory basis of extraterritorial jurisdiction -National contacts test *14th amendment applies to the state, 5th amendment applies to federal -National contacts is not enough, they must also consider the burden on the defendant A. Fairness to consider factors such as: 1. Extent of the defendants contacts with the particular forum 2. The inconvenience of defending in that forum 3. Judicial economy 4. The probably locus of discovery 5. The interstate nature and impact of the defendants activities B. Pure national contacts analysis: due process is satisfied if the defendant in a federal question action has minimum contacts with the United States and does not require any inquiry into fairness or reasonableness of litigating in a particular judicial district. 1. If the plaintiffs claim does not arise from or relate to the defendants national contacts, the plaintiff must show that those contacts were sufficiently continuous and systematic to justify the exercise of general jurisdiction. C. Foreign Sovereigns 1. Not a person within the meaning of Due Process Clause of the Fifth Amendment

2. Congress is free to vest jurisdiction over the person of a foreign state in the federal courts without constitutional constraint and it has provided for worldwide service of process upon such defendants 3. Doctrine of sovereign immunity: a rule of international law under which sovereign states and their instrumentalities are generally immune from suit in the courts of other nations Notice and Service of Process

Chapter 3: Providing Notice and an Opportunity to be heard


A. The Constitutional Requirement of Notice 1. Even with sufficient contacts as in International Shoe, there is an independent due process requirement that the state must afford the defendant adequate notice of the proceeding and a fair opportunity to appear and protect his interest. 2. Pennoyer v. Neff: Only personal service of process was sufficient. Service by publication in a newspaper is not sufficient. *Service upon an agent appointed by the party is equivalent 3. When something less than personal service of process is utilized to give notice, it will scrutinize the practical likelihood that the defendant in the particular case will receive actual notice, even though the means used may be likely to give such notice in most cases. 4. Greene v. Lindsey: posting on an apartment door is not sufficient 5. Covey v. Town of Somers: notice was insufficient when given to a defendant known to have been adjudicated incompetent and institutionalized 6. When method used is not personal service, courts take a fact specific look to determine if the method is likely to give notice -Only the substitute that is most likely to reach the defendant is constitutionally acceptable The Constitutional Requirement of Notice -Pennoyer was concerned about notice *Simply publishing in a newspaper opens up for fraud -Emerged as a separate and independent requirement of due process *The court has attached great weight to this -Greene v. Lindsey: posting notice on the apartment was not sufficient because there were children in the building that regularly tore the notices off the building -McDonald v. Mabee: Only the substitute that is most likely to reach the defendant is acceptable Mullane v. Central Hanover Bank & Trust Co. P. 183 Supreme Court of the United States, 1950

339 U.S. 306, 70 S.Ct. 652, 94 L.Ed. 865 Facts: In January 1946, Central Hanover Bank and Trust Company established a common trust fund in accordance with the New York Banking Law. In March 1947, it petitioned the Surrogates Court for settlement of its first account as common trustee. The only notice given to the beneficiaries was publication in a local newspaper for four successive weeks. A special guardian was appointed to represent all person known or unknown not otherwise appearing who might have an interest in the trust. Procedure: Appellant objected to the notice and statutory provisions and said the court was without jurisdiction. Overruled by Surrogate. Affirmed by Appellate Division of Supreme Court and Court of Appeals. Issue: Was notice to beneficiaries on judicial settlement of accounts by the trustee of a common trust fund established under the New York Banking law constitutionally sufficient? Held: No, insofar as the published notice are urged on behalf of any beneficiaries whose interest or addresses are unknown to the trustee. Under the circumstances it is not reasonably calculated to reach those who could easily be informed by other means. Analysis: The court first analyzed this case as being in personam, not in rem, because the proceeding may deprive the beneficiaries of property rights. The court said they had to balance the States interests and the individuals interest. The court did not create a formula for the balancing. The court said that notice must be of such nature as reasonably to convey the required information and it must afford a reasonable time for those interested to make their appearance. (reasonably calculated) Publication alone is not a reliable means, unless it is not reasonably possible or practicable to give more adequate warning. Mullane v. Cantral Hanover Bank & Trust -Proceeding in the surrogates court (probate court, orphans court): administration of estates and trust -Common trust fund legislation -Notice given for four weeks in a New York publication -In personam because this was adjudicating the personal rights of the defendants *The decree would cut of the beneficiaries rights to sue after the court certified the accounting of the trust. It denies them the surcharge if the accounts had been mismanaged -The court said it really didnt matter if it were in personam or in rem -If this could not be done in New York, there was no where else it could be done. -Notice reasonably calculated under all of the circumstances to reach the intended audience

-Publication was okay for such beneficiaries whose identity and contact information was unknown -Though they are not reaching the unknown beneficiaries, they are reaching other people who have the same interests -The form should not be substantially less likely to reach them B. Service of Process 1. Process: papers which are served on the defendant to give him notice of the commencement of a lawsuit -Summons and a copy of the Complaint 2. Two functions: -Act by which the courts jurisdiction attaches to the defendant -Supply notice in a manner and at a time that afford the defendant a fair opportunity to answer the complaint and present defenses and objection 3. Primacy of personal service: optimum method for service of process -hand D service personally anywhere in the forum state. 4. Substituted or constructive service of process: other than in-hand service -Must at least be reasonably calculated under all the circumstances to give actual notice to the defendant -Must be at persons dwelling and someone who resides there must be there. 5. Quash: when service of process is improper. The court sets aside service but does not dismiss the Complaint, so that the plaintiff can then effect proper service 6. Federal Rule 4(d): Waiving Service- mail the process and waiver form to defendant in regular first class mail, and have to put in return addressed and stamped envelope, and Defendant can sign and mail back waiver form and P files that with court. -If the D fails to return the waiver form (30 days), will formally serve and P will have to pay cost of service. Maryland State Firemans Association v. Chaves P. 199 United States District Court, District of Maryland, 1996. 166 F.R.D. 353. Facts: MSFA sued John Chaves, President of the FireFighters Association of America. They served him with the summons and complaint through first class mail. MSFAs attorney received three phone calls from a representative of the FFA. He did not hear from anyone after that. Procedure: Motion for judgment by default. Granted for defendants failure to plead. Entry of default set aside. Issue: Was the service of process invalid? Held: Yes.

Analysis: The summons did not meet the requirements of the Maryland Rule of Procedure because it was sent by first class mail, not certified mail. 7. Rule 4(e)(2)(B): Service on a Person Residing in Defendants Dwelling or Usual Place of Abode 8. Rule 4(e)(2)(C): Delivery to an Agent Authorized by Appointment -There must be evidence that the defendant intended to confer such authority upon the agent National Equipment Rental, LTD. v. Szukhent P. 205 Supreme Court of the United States, 1964. 375 U.S. 311, 84 S.Ct. 411, 11 L.Ed.2d 354. Facts: The petitioners claimed the respondents defaulted under a farm equipment lease. Two copies of the summons and complaint were delivered to Florence Weinberg. She mailed them to the respondents. Procedure: District court quashed service of summons. Court of appeals affirmed. Reversed and remanded. Issue: Whether the person upon whom the summons and complaint were served was an agent authorized by appointment to receive the same, so as to subject the respondents to the jurisdiction of the federal court in New York. Whether a party to a private contract may appoint an agent to receive service of process within the meaning of Federal Rule where the agent is not personally known to the party and where the agent has no expressly undertaken to transmit notice to the party. Held: Yes. Analysis: Justice Black dissented along with Justice Brennan. 9. Federal Rule 4(h): Serving a Corporation, Partnership, or Association -Insurance Co. of North America v. S/S Hellenic Challenger: Adjuster who had accepted service was not expressly authorized to do so. The adjuster misplaced the summons and complaint. The court denied the defendants motion to set aside the judgment. The plaintiffs service was reasonably calculated. -Fashion Page, LTD. v. Zurich Ins. Co.: Executive secretary was an agent authorized by appointment -Serve an officer, or a managing or general agent. (looking for someone with enough job responsibility that expect it would be okay and transmit important paper) -4(E)(1)- allows to use state methods, so can use any state rules if available. 10. Rule 4(f): Serving an Individual in a Foreign Country -Hague Convention: A requirement that each Contracting State establish a Central Authority, which will receive and execute requests for service from judicial authorities in other Contracting States, and

will see that a certification that service has been effected is returned to the court of origin. -Volkswagenwerk Aktiengesellschaft v. Schlunk: Convention applies only if service actually is made abroad 11. Return of Service C. Immunity from Process 1. Nonresident litigants, witnesses, and attorneys are immune from service of process when they are in the forum solely for the purpose of participating in a judicial or quasi-judicial proceeding, and also for a reasonable time to enter and leave the forum. 2. Sivnksty v. Duffield: a criminal defendant is immune from civil process if he is voluntarily within the jurisdiction but not if he is involuntarily detained pursuant to arrest or extradition. 3. Wyman v. Newhouse: a judgment procured fraudulently lacks jurisdiction and is null and void. 4. Tickle v. Barton: The service of process, having been procured by trickery, deceit and subterfuge is null and void and of no force of effect. -The court may have jurisdiction but may choose not to exercise it. Service of Process -Function of Process: *Giving of notice *Means by which the courts jurisdiction attaches to the person of the defendant -Maryland State Firemens Assn v. Chaves *Chaves did not return the waiver (it does not appear that he was sent it) *The old rule said that if it could be proven the defendant received the process then the waiver did not have to be send *Since Chavez did not return the waiver, he had to be served with process, which he was not -4k1: Where you can serve -4e1: How to serve: if using state laws, the federal court essentially becomes another court in the state -If you are Chavezs attorney, what do you do now? *Reserve the process correctly: serve by certified mail -What if the statue of limitations ran by the time process was served properly? *Action is deemed to be commenced when it is filed with the court (it would have been okay in these majority jurisdictions) *A minority of jurisdictions say an action is not deemed commenced until the process is served on the defendant (it would not have been okay in these jurisdictions because the limitation would have run) -National Equipment Rental *Is Weinberg an authorized agent? Yes.

~The district court said the contract did not require her to promptly forward the summons and complaint to the defendants and thus it was invalid ~Wuchter dealt with statutory limitations and thus the due process clause was brought up. This case was not statutory, it dealt with a contract between two parties and there is no due process issue *What body of law should be looked to? Federal law, New York law, or Michigan Law ~The majority did not decide. ~The dissent said it should be decided under New York law under which this would not be a valid appointment *Jurisdiction by consent *Differences between the majority and the dissent are policy issues Challenges to Jurisdiction Over Persons and Property A. Special appearance: procedure at common law by which a defendant presented a challenge to the courts exercise of personal jurisdiction without submitting to the courts jurisdiction for any other purpose. B. Baldwin v. Iowa State Traveling Mens Assn: a defendant who makes no appearance whatsoever remains free to challenge a default judgment for want of personal jurisdiction. C. Limited-Appearance: allows a defendant in an action commenced on a quasi-in-rem basis to appear for the limited purpose of defending (on the merits) his interest in the attached property without submitting to the full in personam jurisdiction of the court. D. Final Judgment rule: only a final judgment of the trial court is appealable E. Final order: one which ends the litigation on the merits and leaves nothing for the court to do but execute the judgment. Challenges to Jurisdiction -Question 1 *Yes, he can ignore it. But a default judgment will probably be entered against him. The plaintiff would then have to take the judgment to PA and ask the court to give full faith and credit to the judgment. Then, the lawyer can collaterally attack the judgment and would violate due process. ~If the PA court determines the NY court did have jurisdiction, then he can no longer argue on the merits and he will no longer be able to present his forgery defense ~Not a clear question. The court may have jurisdiction even with a fraudulent service of process but may choose not to exercise it. -Question 2

*He can make a special appearance to contest the courts jurisdiction, assuming that New York has a special appearance clause (Every state has a common law special appearance) -Question 3 *Once you have submitted the jurisdictional question to the NY court, you cannot walk away from the judgment that they have jurisdiction -Question 4 *Not to litigate on the merits, allow a default judgment, and then appeal that judgment in the NY court system *But you have waived any right to litigate on the merits -Final Judgment rule: only appeal final judgments *1292 provides for exceptions *Some states do not follow the rule *Why do courts insist on finality? To prevent multiple appeals from the same case -Question 5 *Assert the jurisdictional defense. ~But when he argued the case on the merits he waived any objections to jurisdiction ~He could argue that he never had a choice after they disagreed with him on the jurisdictional issue ~In the federal courts and a vast majority of the states, if you argue the jurisdictional defense and lose, and then argue on the merits, you have not waived the jurisdictional defense in the appellate courts -Question 6 *He can still ignore it, but if the default judgment is entered then he is going to lose the $250,000 or as much as the law suit is for *The bank account is totally unrelated to the claim against him (Shaffer) *Limited-appearance -Question 7 *The full faith and credit option (ignore it and allow a default judgment and then collaterally attack it. *Rule 12b: filing a motion for lack of personal jurisdiction ~Assert the defense the responsive pleadings (the answer) *Special appearance not required in the federal courts *Rule 12g: The defenses were available at the outset and can not be claimed in another motion. But he didnt make an earlier motion, he filed an answer. *Rule 12hb2: asset defenses in a preliminary motion or a responsive pleading or in an amendment to a responsive pleading as a matter of course (do not need permission, the court grants this right) ~If not done this way, you waive the rights to defenses

*He can not make the amendment because he must have done it within 20 days ~15a1b: he wants to amend his answer to which no responsive pleading is allowed *He waived the defenses *Threshold defenses: raised at the outset of the process. If not raised at this point they are lost. Subject Matter Jurisdiction A. Subject Matter Jurisdiction Generally 1. Court of general jurisdiction: have jurisdiction over all civil matters except to the extent that certain kinds of actions may be carved out of that jurisdiction and entrusted to the exclusive jurisdiction of another court. 2. Function of specialized courts -Deal with particular subject matter -Deal with amount of money or property in controversy Subject Matter Jurisdiction (over the case, claims) -Type of controversy -Congress did not have to establish lower Federal Courts. The Supreme Court is the only one required. -Nine categories and statutory basis. Act of Congress inferring jurisdiction on the federal courts to hear the case -Federal Question Jurisdiction, Diversity Jurisdiction B. Subject Matter Jurisdiction in the Federal Courts 1. In order for a federal court to decide a case, not only must it fall within one of the categories enumerated in Article III, Section 2, but Congress must have conferred jurisdiction on the court by statue -Not exclusive, most cases can be brought in state courts as well -Exclusive only if Congress has clearly demonstrated intent to make it so 2. Two types: federal question jurisdiction, diversity jurisdiction -Federal question: arising under the Constitution, the Laws of the United States, and Treaties made, or which shall be make under their authority -Diversity: between Citizens of different states. - case is between citizens of different states -amount exceeds $75,000. C. Federal Question Jurisdiction 1. Article III Section 2 of the U.S. Constitution. -All cases, in law and equity, arising under this Constitution, under the laws of the United States, and Treaties made, or which shall be made under their authority.

2. Osborn v. Bank of the U.S.: Bank of US brought suit in a federal court to prevent Ohio from collecting a tax it believed to be unconstitutional. The state auditor forcibly entered the bank and took the money anyway. The court had subject matter jurisdiction because the bank alleged the tax violated the U.S. constitution. The validity of the contract depends on the law of the US. Louisville & Nashville R. co. v. Mottley P. 299 Supreme Court of the United States, 1908 211 U.S 149, 29 S.Ct. 42, 53 L.Ed 126 Facts: Husband and wife (Mottley), of Kentucky, brought suit in the district court against a Kentucky railroad company alleging their negligence lead to the plaintiffs injuries. The alleged a breach of contract. In exchange for releasing the railroad from liability for injuries, the railroad would give them free transportation. They stopped after a few years. Procedure: Defendant demurred. Circuit court overruled. Appeal. Reversed, ordered to be dismissed for lack of jurisdiction. Issue: Whether the act of Congress of June 29, 1906 makes it unlawful to perform a contract for transportation of person who, in good faith, before the passage of the act, had accepted such contract in satisfaction of a valid cause of action against the railroad. Whether the statue, if it should be construed to render such a contract unlawful, is in violation of the 5th Amendment of the Constitution. Held: The court did not have to consider these questions because the court below did not have subject-matter jurisdiction. Analysis: There was no diversity of citizenship in this case. The only ground of jurisdiction could have been that the suit arose out of the Constitution or laws of the U.S. The court said this would likely arise in the course of litigation but that the original cause of action did not arise out of the constitution. A federal question simply arising in the course of the trial is not enough to assert jurisdiction. Federal Question Jurisdiction -If there was no statutory basis, the suit had to be brought in state court even if it was regarding a federal jurisdiction -Just because an action can be brought in federal court does not mean it has to be brought there *Congress can make exclusive jurisdiction -In 1981 the jurisdictional amount was deleted from the federal question jurisdiction -Louisville & Nashville v Mottley *They did not have subject matter jurisdiction *The Supreme Court raised the jurisdictional issue on its own

*The railroad said it terminated the contract because it was required to do so because of a newly enacted statue. The Mottleys said they read the statute wrong and if it were read correctly it would be an unconstitutional taking of their property *Issue of structure and constitutionality of federal statute *Must be a violation of a right arising out of federal law, not simply a defense using federal law *The Mottleys cannot anticipate that defense in their complaint (wellpleaded complaint) ~It was a simple breach of contract case that would be an issue of state law *For subject matter jurisdiction, it must arise on the face of the complaint and must be a well pleaded complaint (only elements of plaintiffs complaint, cannot anticipate federal law defense or counterclaim) 3. Artful pleading (well pleaded case): an attempt by the plaintiff to create federal question jurisdiction through the anticipation and inclusion of a federal defense on the face of its complaint in an action brought under the Declaratory Judgment Act. D. Diversity Jurisdiction (Diversity of Citzenship) 1. Allows the federal courts to hear cases in which the claims arise solely under state law, so long as constitutional and statutory requirements are met 2. Complete diversity: no diversity jurisdiction if any plaintiff is a citizen of the same state as any defendant, no matter how many parties are involved in litigation -Citizenship is state of domicile. (Mas v. Perry) -To change: need to physical place, and intent to make it new home. 3. Minimal diversity: when at least one plaintiff is a citizen of a state different from that of at least one defendant 4. Corporate Citizenship: a corporation is a citizen for purposes of diversity jurisdiction, but what state? 1331(c)(1) -Nerve center test (Hertz) : the court looks to the location of the corporations home office or executive headquarters, the nerve center from which it radiates out to its constituent parts and from which its officers direct, control, and coordinate all activities -Corporate activities test (operating asset test): the court looks to the state where the bulk of the corporations productive activities are carried on -Total activity: courts consider the total activity of the company considered as a whole -Generally a citizen of: the state(s) in which it is incorporated and (2) the state in which it has its principal place of business

5. Unincorporated Associations: deemed to take on the citizenship of each of its members -Partnerships, labor unions, charitable organizations 6. Determining citizenship Mas v. Perry P. 256 United States Court of Appeals, Fifth Circuit, 1974 489 F.2d 1396, certiorari denied 419 U.S. 842, 95 S.Ct. 74, 42 L.Ed.2d 70 Facts: Jean Paul Mas, a citizen of France, and Judy Mas were married at her home in Jackson, Mississippi. They pursued graduate studies at Louisiana State University in Baton Rouge, Louisiana. Then they moved to Park Ridge, Illinois. They intended to return to Baton Rouge. They rented an apartment from Oliver Perry, a citizen of Louisiana. They sued him after discovering their apartment contained two way mirrors and they had been watched by Perry. Perry claims the district court lack subject matter jurisdiction because there was no diversity of citizenship and the jurisdictional amount was not met. Procedure: Verdict for plaintiffs. Motion to dismiss for lack of jurisdiction. Denied. Appeal for lack of jurisdiction. Affirmed. Issue: Was there diversity of citizenship in the case? Held: Yes. Analysis: The court concluded that Mr. Mas was a citizen of Mississippi for diversity purposes and since Perry was a citizen of Louisiana, there was diversity for that claim. The court reiterate the idea of complete diversity. They said that diversity must be present at the time the complaint is filed and it is unaffected by changes in the citizenship of the parties afterwards. For purposes of diversity, the court said citizenship means domicile. Mere residence in a state is not sufficient. The court defined domicile as the true, fixed, and permanent home and principal establishment, and to which he has the intention of returning whenever he is absent therefrom. It is generally held that a husband and wife have the same domicile. *The party asserting diversity jurisdiction has the burden of proving its existence Diversity Jurisdiction -Question 1: Yes, typical diversity jurisdiction case. *What makes someone a citizen of a particular state? -Question 2: Yes, diversity jurisdiction still exists. For diversity purposes there is no difference between the two situations. Other rules then come into play to determine which one or more of the district courts the case can be pursued in. *Is this a proper venue? Yes, 1391a2. The accident occurred in Philadelphia which is in the Eastern District of PA. Must also be

able to establish in personam jurisdiction (is there a long arm statute) *Three things must be satisfied: Subject Matter Jurisdiction, Proper Venue, In Personam Jurisdiction -Diversity jurisdiction exists to prevent (the possibility of) local bias. But if the plaintiff is bringing the suit in a federal court in his own state, when he could also bring the suit in a local court, then what is the rationale? -Question 3: No, D is a corporate citizen of Delaware and New York so it is a dual citizen. *Requirement of total diversity: no plaintiff can be a citizen of the same state as any defendant *In special areas, there can be minimal diversity *This will depend on whether the New York Court finds that D has its principal place of business in NY. It has its executive offices, but this does not mean that it is its principal place of business. ~Three separate tests: Nerve center, corporate activities, total activity ~Nerve: Yes. There would be no diversity of citizenship -Exceptions: probate disputes, domestic relations disputes -Historical justification: bias against out of state litigants *Is this still an issue today though? There does not seem to be a general congressional will to abolish it. But they did impose some limits. -Question 4: No, the legal representative of a minor is deemed to be a citizen of the same state as the minor. When this case was decided, 1332c was not in place. *This would allow someone to manufacture jurisdiction, since they could just appoint someone from out of state *Section 1359 does not allow manufacturing jurisdiction *Studies showed that damage awards were higher in district courts at the time, so the lawyer obviously wanted to litigate in the district court instead of the court of common pleas -Alienage jurisdiction: between citizen of state and citizen of a foreign state- no jurisdiction in actions between aliens *Kramer v. Caribbean Mills: no valid business reason. It was made solely to manufacture jurisdiction where it would not otherwise have existed *Fact specific to each case why certain appointments are made: that is why 1332 c2 was made -Mas v. Perry *Question 1: Domicile *Question 2: Jean Paul was not a Louisiana domiciliary because he was only in Louisiana as a student and lacked the intent to remain there indefinitely, he was also not a citizen of the U.S. ~Physical presence must concur with requisite state of mind (intent to make the place their home)

~Certain classes of people have different rules (military, students): assumption that there is no mental intention to stay there in the case of students *Must be a citizen of the United States before being considered a citizen of any state (Jean Paul was a citizen of France) *Instead, he was considered domiciled where his wife was domiciled. Domicile of origin was France. ~If Judy was not in the picture, then Jean Paul could sue under alienage jurisdiction *Question 3: If he was a domiciliary of France, she would become a domiciliary of France. They would not have had jurisdiction under this. If she were a domiciliary of France, she would not be a domiciliary of any US state. She could also not sue under alienage because she is not an alien. (The US court would not consider her a citizen of France, regardless of what French law states) ~For purposes of diversity, only American citizenship is recognized. Dual citizenship is not. ~Under traditional rules as above, when they sue together, the court does not have total diversity *Question 4: Any American citizen who is domiciled abroad *Question 6: It would depend upon where Mr. Mas was domiciled. It would also depend if he intended to take up permanent residence in the U.S. (was he admitted by the government for permanent residence? Was it just a student visa? 7. Diversity Jurisdiction (The Jurisdictional Amount Requirement) -Exceeds $75,000, exclusive of interest and costs -The plaintiff has the burden of persuading the court that the jurisdictional amount is truly in controversy -Aggregation add claims together to exceed $75,000. 8. Supplemental [Pendant and Ancillary] Jurisdiction Federal Ct hear claim that otherwise wouldnt hear. -Plaintiff has multiple claims against a defendant, some of which are federal claims within federal question jurisdiction and others are state-law claims outside of the jurisdiction -Plaintiff has multiple claims against different defendants, all related to the same disputed transaction, but only some have an independent basis for federal jurisdiction -Multiple plaintiffs may have multiple claims against multiple defendants, all related to the same disputed transaction, but only some of the claims have an independent basis for federal jurisdiction -Pendent jurisdiction: plaintiff appends a claim lacking an independent basis for federal jurisdiction to a claim possessing such a basis

*Pendant party: when a plaintiff files a federal claim against one defendant and a state-law claim against a different non-diverse defendant - Ancillary jurisdiction: either a plaintiff or a defendant injects a claim lacking an independent basis for federal jurisdiction by way of a counterclaim, cross-claim, or third-party complaint United Mine Workers of America v. Gibbs P. 300 Supreme Court of the United States, 1966 383 U.S. 715, 86 S.Ct. 1130, 16 L.Ed.2d 218 Facts: Paul Gibbs (TN) was awarded compensatory and punitive damages against United Mine Workers of America (TN) for alleged violations of Section 303 of the Labor Management Relations Act, 1947 and of the common law of Tennessee. Gibbs lost his job and never entered into performance of his haulage contract. He also began to lose other trucking contracts and mine leases held in nearby areas. He said these were as a result of a concerted union plan against him. Procedure: Verdict for plaintiff. Appeals court affirmed. Certiorari granted. Reversed. Issue: Whether the District Court properly entertained jurisdiction of the claim based on Tennessee law. Held: Yes. Analysis: The court in Hurn v. Oursler held that state law claims are appropriate for federal court determination if they form a separate but parallel ground for relief also sought in a substantial claim based on federal law. Federal courts may decide to dismiss cases upon the nonfederal ground, but not upon the nonfederal cause of action. This court said that federal claims must have substance sufficient to confer subject matter jurisdiction on the court. The state and federal claims must derive from a common nucleus of operative fact. If plaintiffs claims are such that it would be expected for them to be tried in one court, then if there are substantial federal issues, there is power in the federal court to hear all of them. Supplemental Jurisdiction -To what extent can a federal court exercise jurisdiction over a state law claim in the absence of diversity, if it is related to another claim within the federal courts jurisdiction. -United Mine Workers of America v. Gibbs *Taft Tartly act outlaws secondary boycott *Unincorporated citizenship: citizen wherever its members are citizens *He had a federal statutory claim and a relate state law claim arising out of the same alleged conduct but he did not have diversity of

citizenship. If he had brought an action only for the state law claim, the federal court would not have had jurisdiction. But, he asserted it in a lawsuit with the federal question. *What does the nature of the relationship have to be between the jurisdictionally sufficient claim and the state law claim before invoking supplemental jurisdiction? ~Assume Hurn v. Oursler is still the law. B buys power mower and operates it. It goes berserk and amputates Bs right foot. It also chews up the shoe and trouser leg. B brings action in federal court against manufacture of power mower for negligence and strict liability. Is there supplemental jurisdiction? If it is a single cause of action, the court may dispose of the entire case and find for the plaintiff on the state law claim. If there are separate causes of action, then they cannot exercise jurisdiction over the state law action. ~Depends on how you define cause of action: Question is whether the plaintiff suffered one or more actionable wrongs. Was there an invasion of separate and distinct legal rights (two or more causes of action)? ~Right to be secure from negligent injury in person and secure from damage to property, this could be argued. *Claim is the Federal Rules equivalent of cause of action *Did the state and federal claims arise from a common nucleus of operative fact? (pg 328) Such that ordinarily the plaintiff would litigate them all in one lawsuit, then yes, supplemental jurisdiction can come into play. ~It is much more efficient, more factual approach *If Gibbs could not pursue both, he would have had to give up one claim or his would have had to bring two different lawsuits. Or he could have brought both claims in state court. *Single constitutional case? -Common nucleus of operative fact Statute 1367 -Legislative history: determine legislative intent -The statute and the legislative history both point to Gibbs. They intended to adopt the Gibbs test. (Common nucleus of operative fact) -Question 2a: Yes, the legislature codifies it -Question 2b: Yes. If we just had subsection A, yes. It embodies the constitutional test in Kroger. But there must also be a statutory basis (1367) -1367B speaks of excluding certain kinds of claims. -Kroger would still be good law. Kroger made a claim under Rule 14. -If passed under section A, there must be something under B to exclude it. Otherwise, it passes. If Rule 14 was deleted from B, then Kroger

would have been reversed and the court would have had supplemental jurisdiction. -Section B deals only with diversity cases. -Question 2c: If only A, yes there would be supplemental jurisdiction. Is it excluded by B? No. The action of A is a federal question action so the limitations of 1367 B do not apply. The statue overrules Finley -Question 2d: No for the same reasons as above. It was a federal question case. The state law claim was closely enough related to satisfy subsection A. -Question 3: Does rule 14 authorize this? Yes, as long as Cs claim arises out of the same claim as A. Subsection A is satisfied. Does subsection B exclude supplemental jurisdiction? No, it does not because it only excludes claims by plaintiffs. It does not exclude any defendant claims. *Defendant claims are treated differently. -Subsection C: Doctrine of judicial discretion. -Aldinger v. Howard: Did not allow pendant party jurisdiction even if the claims derived from a common nucleus of fact. -Finley v. United States: Did not allow pendant party jurisdiction. See 135 M for similar facts. Plane is supplemented with a car. Owen Equipment and Erection Company v. Kroger P. 138 M Supreme Court of the United States, 1978 437 U.S. 365, 98 S.Ct. 2396, 57 L.Ed.2d 274 Facts: James Kroger was electrocuted when the boom of a steel crane he was walking next to came to close to a high-tension electric power line. His wife, a citizen of Iowa, filed suit against the Omaha Public Power District, a Nebraska corporation. OPPD filed a third-party complaint against Owen, whose principal place of business was in Iowa, because they owned and operated the crane. Procedure: Action for wrongful death. OPPD granted summary judgment. Owen moved to dismiss for lack of jurisdiction. Not granted, verdict for plaintiff. Affirmed on appeal. Reversed. Issue: In an action in which federal jurisdiction is based on diversity of citizenship, may the plaintiff assert a claim against a third-party defendant when there is no independent basis for federal jurisdiction over that claim? Held: No. Analysis: Undisputedly, there is no independent basis for federal jurisdiction of the plaintiffs state-law tort action against Owen. This court said the Appeals court failed to understand the scope of the doctrine of the Gibbs case. The court discussed the idea of complete diversity, which is lacking in this case and was lacking from the beginning. The court said that the nonfederal claim in this case was not ancillary to the federal one. Also, the non federal claim was asserted by the plaintiff, who voluntarily chose to bring suit upon a statelaw claim in federal court.

Justice White and Brennan dissented. They said that this plaintiff did NOT assert a claim against a third-party by adding the new party to the lawsuit. The original defendant brought Owen into the suit, and therefore, the plaintiff sought to assert a claim arising out of the same transaction that was already before the court. -Owen Equipment and Erection Company v. Kroger *It would appear Mrs. Kroger v. OPPD v. Owen *Question 1: Yes, it allows her to do this. Rule 14a3: any claim arising out of the transaction or the occurrence that is the subject matter of the plaintiffs claim against the third-party plaintiff. The court did not allow her to do this because the Federal Rules of Civil Procedure cannot extend on contract the reach of federal jurisdiction. The rules assume that the court has subject matter jurisdiction. *Question 2: Yes, it came from the same nucleus of operative fact. *Question 3: The outcome would not have been any different had the summary judgment for OPPD been denied. Even satisfying the Gibbs test would not be enough. Gibbs established the constitutional outer limits. There must also be a statutory basis which in this case is section 1332a1 which involves diversity jurisdiction. *Question 4: The dissent believes total diversity was only relevant to the original parties brought into the lawsuit. *Question 5: For purposes of supplemental jurisdiction, we treat defendants differently than plaintiffs because defendants are being brought into court against their will. If they could not do this, then they would have to sue the third party separately while litigating the original claim. 9. Removal Jurisdiction: a procedural device which permits the defendant to shift into a federal court a state court action which the plaintiff could originally have brought in federal court but elected to bring in a state court. -Functional test to determine what is a court for removal purposes -D can remove it to Federal court if Federal Question or Diversity of Citizenship. -EXCEPT cannot remove a diversity case if any defendant is a citizen of the forum. Removal Jurisdiction -An anomaly -Section 1441, 1446 -Defendant files a notice of removal of the action

*1446d: send notice to all adverse parties and file copies with clerk of state court which shall effect removal and the state court will proceed no further until the federal court remands *Not a motion, simply a notice. No federal judge sees this until the case has been removed. The defendant does not need permission to effect removal. *Question of whether it is a removal case arises after when and if the plaintiff files a motion to remand (1447) or if the judge on his own initiative raises the question of remanding -Question A: Federal questions can be removed without regard. Any other actions (diversity) can not be removed if the defendant is a citizen of the state in which the action is brought. 1441b. The defendant is a corporate citizen of New York, so this is not a removable action. *1447c: Motion to remand must be made within 30 days of notice of removal. The plaintiff waited 6 weeks, so he waived his right to remand. This is assuming it was not based on a lack of subject matter jurisdiction. -Question B: 1441b- a federal question so the motion to remand should not be granted. -Question C: 1447d-an order remanding a case to the State court from which it was removed is not reviewable on appeal (except in civil rights cases). It does not matter how wrong the remand may be. *Basic issue: is a forum selection clause a proper basis for remand even though it does not fall under the language of 1447c *Thermatron: Supreme Court imposed a significant limitation on 1447d in holding that it only applies to remands grounded in accordance with 1447c. *Is the remand on 1447c grounds? If it is, then it is immune from review. If it is not, then it can be reviewed. *Arguing in favor of 1447c: yes, there is a defect other than a lack of subject matter jurisdiction. Any basis for remand has to be lack of subject matter jurisdiction or anything else. There would be nothing to apply D to, because C occupies the universe. All remands would be based on 1447c. *Kamm v. Itex: not a defect. Not all remand orders are immunized under the current language because 1447c speaks of remands for lack of subject matter jurisdictions and a defect other than a lack of subject matter jurisdiction. Is there a third category? Why is a forum selection clause a defect? Hence, it is reviewable and the 30 day requirement does not apply. Defect would be a failure to comply with the statutory requirements for removal. *What remands fall into the reviewable category? Forum selection clauses are not defects. Something extrinsic to the provisions required by statutes.

-Question D: The case may be removed by the defendants (Requirement of unanimity of the defendants, they must both remove). The case was improperly removed in the first place. This would be a defect because it is a failure to comply with a statutory requirement. The remand was also untimely. *No subject matter jurisdiction problem *Was this a proper remand? No, it was violating an express statutory limitation on its power. It was beyond the 30 day time limit. *Can the district court remand the case on its own initiative? No. It does not have the authority to remand without a timely motion. -Question E: Would have to know where D principals place of business is. P cannot sue D if principal place of business was either PA or NY. -Question F: Venue A. Venue Generally 1. Regulates the geographical allocation of business within a judicial system 2. Transitory actions: can be brought in any county where the defendant can be found (served with process) 3. Local actions: i.e. actions involving property (real property) are required to be brought where the property is situated. 4. Venue is statutory B. The Local Action Rule Venue in the Federal Courts -Exception to the rule that venue is statutory: Local action ruleusually land, sometimes chattels. *Core local action cases: disputes as to conflicting interests in claims of land -Question 1: 1391(a) founded only on the diversity of citizenship. The S.D.N.Y would not be a correct venue so the motion should be granted. *There is not residential venue because none of the defendants reside in NY -If multiple defendants all resident in different areas throughout a state, may file claim against all in one of the courts where one lives. *There is not a transactional venue because nothing occurred in NY *It is arguable that there would be personal jurisdiction over C, but this is an unrelated claim which would make it difficult to obtain jurisdiction over C in this lawsuit, and there is another district

where the action may be brought (US District Court for the District of Maryland, fall back venue) *1391(a2) has to do with federal question issues. -Question 1(c): Does Maryland have a long arm-statue that would reach the out of state defendants? Yes. The District Court for the District of Maryland would be a proper venue, but you would need to get in personam jurisdiction which could be obtained in this case. *Must have subject matter jurisdiction, must have proper venue, and must be able to get in personam jurisdiction over the defendant *If B was a resident of Scranton, he could bring it either the middle district or the eastern district of PA. 1391(a1). *Suppose it appears that C has a depot in NY where he keeps his trucks and B picked up the truck there. Would this satisfy 1391(a2)? Maybe. What if C negligently maintained his truck there? Then it would definitely be satisfied. -Question 1(d): No residential venue under 1391(a1). No transactional venue under 1391(b) because it occurred in Canada. There may be fall back venue, because there is no other federal judicial district in which the action may be brought. *By and large, the only scenario that will fit fall back venue is when the events occurred in no federal judicial district (outside of the country) and in situations where the defendants do not reside in the same state. *C would be subject to in personam jurisdiction in the E.D of PA because he resides in PA. He would have to be subject to it when the action was filed with the court (the action was commenced). However, B is domiciled in NJ but he regularly visits his niece in Philadelphia. He is subject to in personam jurisdiction because he was able to be served with process while in Philadelphia. But was he subject to it when the action was commenced? -Question 2: No residential. No transactional. If D has minimum contacts with the E.D. of PA then fall back venue may apply. 1391(a3). It would be deemed a resident for venue purposes 1391(c). *When there are multiple districts, must determine if the corporation is subject to in personal jurisdiction in the state. Then, you must break off the districts and view them as separate states. *Must know what in fact Ds contacts with the Eastern District of PA are. *P could bring the suit in Delaware. -Question 3: Leroy was a federal question case under 1391(b). There is no change as to residential venue. What about transactional? The effect of what the Idaho defendants did was felt by the defendants at their headquarters in TX. The offer was tendered there. But the impact is still the only thing that happened in Dallas.

*Congress did not intend to provide for venue at the residence of the plaintiff, but arent there going to be a large class of plaintiffs who feel the impact of the defendants negligent conduct where the plaintiff is located? *Say the answer is no, the amendment does not change the result. Can it invoke fall-back venue? No, because Idaho was an appropriate venue so it would be hard to say there was no another appropriate district. *If the New York and Maryland officials were still defendants, then fallback venue could probably be invoked since there would be no other district. There is no residential or transactional, and there would be no long-arm statues to cover the two defendants in the other states. *Found typically meant found and physically served with process in the jurisdiction (probably an oversight) -The transferee must be a proper venue and have personal jurisdiction over the defendant. Must be true without waiver independently true. Reasor-Hill Corp. v. Harrison P. 349 Supreme Court of Arkansas, 1952 220 Ark. 521, 249 S.W.2d 994 Facts: The suit was brought by the Planters Flying Service to collect an account for having sprayed insecticide on Bartons crop in Missouri. Barton said the flying service had damaged his growing crop (he refused to pay) and in his cross-complaint he sought damages from the petitioner for negligence in putting a chemical unsuited for spraying on the market. The petitioner is an Arkansas corporation that manufactures insecticides. They are not authorized to do business in Missouri. Procedure: Motion to dismiss cross-complaint. Overruled. Application for prohibition. Denied. Issue: May the Arkansas courts entertain a suit for injuries to real property situated in another State? Held: Yes. Analysis: The petitioner cannot be served with summons in Missouri, so unless they can be sued in Arkansas they will escape liability completely by staying out of Missouri until the statue of limitations has run. Previous courts would have unanimously held against the respondent. In Arkansas, the statues require that actions for injury to real estate be brought where the land lies. The defendant can be served anywhere in the state. In Livingston v. Jefferson, the court held that a suit for wrongful entry upon land in Louisiana could not be maintained in Virginia. Courts have tended to rely on this case in deciding their own issues. Courts have relied on this reasoning for three reasons.

1. Courts are not in a position to pass upon the title to land outside the jurisdiction. The court said this reasoning makes sense if the issue is between nations. But they said this does not apply to issues between different States. The court said they could easily handle the laws of other states. 2. Since a tort must take place where the land is situated, the plaintiff should pursue his remedy before the defendant leaves the jurisdiction. Again, this makes sense when nations are concerned. However, citizens of various states are allowed to pass freely from one to another and can hardly be expected to file suit that early. 3. There is a reluctance to subject ones own citizens to suits by aliens, especially if the other jurisdiction would provide no redress if the satiation were reversed. Again, this has no bearing when states are involved. The court does not want to provide refuge to those who commit misdoings. The court said the logic has no current logic or equity. Justice McFaddin dissented. He said the Arkansas courts are in no way equipped to handle other states law questions. He said the majority opinion reduced States to the level of mere local administrative units. He said the question of providing sanctuary to those who commit misdoings is a question for the legislature to decide. Reasor-Hill v. Harrison -Majority view is that jurisdictions cannot hear cases for trespass to land in other jurisdictions: still the majority view in federal courts -If Barton could not bring the case in Arkansas, he would have to bring it in Missouri, where the defendant is not subject to personal jurisdiction. Barton would have no forum open to him to pursue his claim against Reasor Hill. -Expanded reach of in personam jurisdiction makes these actions very unlikely -Local action rule is not a venue rule in this case: Arkansas was not deciding where the case could be held in the State. It was deciding whether the state could hear the case at all. *Seems to be a rule of in personam jurisdiction -The rule can be both jurisdictional and venue -Venue defense can be waivable C. Venue in the Federal Courts 1. 28 U.S.C. Section 1391 -District where the defendants reside or where events or omissions occurred that determines venue -Unincorporated association is a resident for venue purposes of any judicial district where it is doing business Bates v. C&S Adjusters, Inc. P. 353 United States Court of Appeals, Second Circuit, 1992 980 F.2d 865

Facts: Bates commenced an action in the Western District of New York against C&S Adjusters Inc. He alleged violations of the Fair Debt Collections Practices Act. Bates incurred the debt while a resident of Pennsylvania. The creditor, a corporation with its principal place of business in Pennsylvania, referred the account to C&S which is a local collection agency that transacts no business in New York. Bates moved to the Western District of New York. C&S mailed a collection notice to Bates at his PA address and it was forwarded to the new address in NY. Procedure: Batess complaint dismissed for lack of proper venue. Appeal. Reversed and remanded. Issue: Whether venue exists in a district in which the debtor resides and to which a bill collectors demand for payment was forwarded. Held: Yes. Analysis: The court looked at Leroy v. Great Western United Corp. where a Texas corporation attempted to take over an Idaho corporation. Idaho officials sought to enforce a state anti-takeover law. The Texas corporation was not allowed to bring suit in Texas against the Idaho officials. The court said the statutory provisions for venue do not focus on whether a defendant has made a deliberate contact, but on the location where events occurred. The court looked to determine if a substantial part of the events giving rise to the claim occurred in the Western District of New York. They concluded that a receipt of a collection notice is a substantial part of the events giving rise to the claim under the Fair Debt Collection Practices Act. The collection notice is located in the Western District of New York. The collection agency did not mark the mailing with a do not forward and they have not objected to the assertion of personal jurisdiction, so trial there would not be unfair. Leroy v. Great Western United Corporation P. 184 M Supreme Court of the United States, 1979 443 U.S. 173, 99 S.Ct. 2710, 61 L.Ed.2d 464 Facts: Idaho has a statue called the Idaho Corporate Takeover Act, which is designed to regulate takeovers of corporations that have certain connections to the State. Sunshine is a target company. Their principal business is a silver mining operation in the Coeur dAlene Mining District in Idaho. They are engaged in business in New York and through a subsidiary in Maryland. Its stock is traded of the NYSE. It is a Washington corporation. Great Western is an offeror. They are a Delaware corporation. They have their executive headquarters in Dallas, TX and corporate offices in Denver, CO. Great Western decided to make a public offer to purchases Sunshine stock for a premium price. Because the offer would cause Great Western to own more than 5% of Sunshines outstanding shares, they were required to comply with the Williams Act and the Idaho Corporate Takeover Act. They filed suit in the Northern District of Texas against state officials in Idaho, New York, and

Maryland responsible for enforcing takeover laws. They claimed the state laws were invalid. Procedure: Claims against NY and MD dismissed. Idaho contested jurisdiction and venue. District court found jurisdiction. Venue improper, but sustained under special venue provision of Securities Exchange Act. Affirmed. Reversed. Issue: Whether the state agents responsible for enforcing an Idaho statue may be required to defend its constitutionality in a Federal District Court in Texas, and if so, whether the statue conflicts with the Williams Act amendments to the Securities Exchange Act of 1934, or with the Commerce Clause of the United States Constitution. Held: No. Venue did not lie in the Northern District of Texas. Analysis: The court said there was no statutory basis for venue under Section 27 of the Securities Exchange Act of 1934. They also said there was no statutory basis under Section 1391(b). They looked at the first test, the residence of the defendants, which is obviously Idaho. The second test is where the claim arose. The appeals court said it arose in Texas. The Court said Great Westerns complaint did not in fact raise justifiable claims against any officials, let alone those in Idaho. The court said this claim has only one obvious locus: the Distinct of Idaho. The bulk of the relevant evidence and witnesses are located in Idaho. The nature of the action challenging the constitutionality of a state statue makes venue there appropriate. D. Forum 1. Gulf Oil Corp. v. Gilbert: Forum non conveniens is simply that a court may resist imposition upon its jurisdiction even when jurisdiction is authorized by the letter of a general venue statue. Courts have the discretion to chance the place of trial on various grounds, such as the convenience of witnesses and the ends of justice. Forum non Conveniens -Gulf Oil v. Gilbert: seminal case. Difference of opinion as to the common law doctrine of forum non conveniens whether it had any application in federal courts. Held that it was applicable to federal courts. It can never apply if there is no jurisdiction in the original court. *It assumes that the plaintiff has played by the jurisdictional and venue rules. It assumes there are two forums available. If there is not another forum available, then the doctrine cant apply. Start with a strong presumption in favor of the plaintiffs choice of forum. *Lays out the kind of factors the court has to weigh in each case to determine whether the defendant has sufficiently rebutted the presumed plaintiff choice of forum. Piper Aircraft Co. v. Reyno P. 365 Supreme Court of the United States, 1981 454 U.S. 235, 102 S.Ct. 252, 70 L.Ed.2d 419

Facts: In 1976, a small commercial aircraft crashed in the Scottish highlands during the court of a charter flight from Blackpool to Perth. The pilot and five passengers were killed. The decedents were all Scottish residents as are their heirs and next of kin. The aircraft was manufactured in PA by petitioner Piper Aircraft. The propellers were manufactured in Ohio by petitioner Hartzell Propeller. The aircraft was registered in Great Britain and was owned and maintained by Air Navigation. It was operated by a Scottish air taxi service, McDonald. The wreckage of the plane was in England. A California probate court appointed respondent Gaynell Reyno, administrator of the estates of the five passengers. She is not related to and does not know any of the decedents or their survivors. Reyno commenced wrongful death actions against Piper and Hartzell in the Superior Court of California. The survivors of the five passengers initiated actions against Air Navigation, McDonald, and the estate of the pilot in the UK. Procedure: Action for wrongful death brought in CA state court. Petitioners (defendant) motion for removal. Suit removed to the United States District Court for the Central District of California. Piper moved to transfer to the US District Court for the Middle District of PA. Hartzell moved to dismiss for lack of personal jurisdiction, or to transfer. Case transferred to Middle District of PA. Piper and Hartzell moved to dismiss for forum non conveniens. District court dismissed. Appeal. Granted. Appeal. Reversed and Remanded. (Supreme Court agreed with District Court) Issue: Did the Court of Appeals err in reversing? Held: Yes. Motion for forum non conveniens is granted. Analysis: The Court of Appeals erred in holding that plaintiffs may defeat a motion to dismiss on the ground of forum non conveniens merely by showing that the substantive law that would be applied in the alternative forum is less favorable to the plaintiffs than that of the present forum. The possibility of a change in substantive law should not be given weight. The District Courts decision regarding plaintiffs choice of forum was appropriate. A foreign plaintiffs choice deserves less deference. Both Piper and Hartzell submitted evidence that they would face evidentiary problems if the trial were held in the US. The District Court was correct in that the problems clearly supported holding trial in Scotland. Finding that the trial in the plaintiffs chosen forum would be burdensome is sufficient to support dismissal on grounds of forum non conveniens. The American interest in this accident is not sufficient to justify the enormous commitment of judicial time and resources that would inevitably be required if the case were tried here. Piper Aircraft v. Rayno *Alternatives: go to Scotland, sue in a state court in PA because they are both subject to in personam jurisdiction there (and local defendant rule, so it could not be removed). Possibly state court in Ohio or another state. If it were brought in a state court in either of those states it will be

irremovable. (However, if you sue in a state court where it can be removed to federal court, nothing will be accomplished) *Can bring forum non conveniens in a state court. They are not required to come to the same conclusion as the US Supreme Court. *Note 4 says that having an alternative forum is a prerequisite to invoking forum non conveniens. ~Piper and Hartzell agreed to submit to jurisdiction in Scotland and agreed to waive any statue of limitations defense that might be available to them ~Cannot say for certain that Scottish courts will entertain the action. *The plaintiffs wanted to be in American courts because Scotland did not recognize strict liability in product liability actions. (Much more favorable to the plaintiff because it relieves him of the task of proving negligence on the defendants part. They only have to prove that the product was defective) ~Also, the damages that can be awarded in Scotland are more limited in wrongful death actions. *Unfavorable change of law may be given weight if the remedy available in the alternative forum is so small that it essentially amounts to no remedy at all. *If the district courts reasoning is reasonable and rational, it must be upheld. Forum non conveniens is highly discretionary. It is reviewable only for abuse of discretion. It is a narrow scope of review. *Too much weight given to foreign party over native party. (p. 398) 2. There must be another more convenient forum where the plaintiff can obtain adequate relief. E. Transfer of Venue 1. 28 U.S.C. 1404(a), 1406(a), 1631 Transfer of Venue -Question 3a: If the court finds either of the claims have merit, instead of dismissing, they should move to transfer the action to another court in the alternative. 1404a. *In Gulf Oil and Piper, the action was dismissed. -Question 3b: 1406a: No--the court could not transfer if 1406(a) was all it had. It must have jurisdiction and it must be in a wrong venue. In 1404(a) it does not need a wrong venue, so yes. Must determine if case could be brought in this district in the first place. *Where venue is improper, it does not just have to dismiss. It can transfer *The court must be persuaded that it is in the interest of justice

*Note 6 on pg 390 Goldawr Inc.v.Heiman granted 1406(a) motion despite not having personal jurisdiction. Hoffman v. Blaski P. 357 Supreme Court of the United States, 1960 363 U.S. 335, 80 S.Ct. 1084, 4 L.Ed.2d 1254 Facts: Blaski, a resident of Illinois, brought a patent infringement action in the US District Court for the Northern District of Texas against Howell and a Texas corporation controlled by Howell. He alleged that the defendants are residents of and maintain their only place of business in Dallas in the Northern District of Texas. Procedure: Defendants moved to transfer to the US District court for the Northern District of Illinois. Granted. Respondents filed a petition for a writ of mandamus direction vacation of that order. Denied. Respondents moved to remand. Denied by Judge Hoffman. Respondents filed anther writ of mandamus direction Hoffman to reverse. Granted. Affirmed. Issue: Whether a District Court, in which a civil action has been properly brought, is empowered by 1404(a) to transfer the action, on the motion of the defendant, to a district in which the plaintiff did not have a right to bring it. Held: No. Analysis: The plaintiffs did not have a right to bring the actions in the respective transferee districts, and therefore the judgments of the Court of Appeals were correct. There was no in personam jurisdiction and it was a wrong venue because it was a patent infringement action which falls under a special venue statue. Justice Frankfurter dissented. He said there can be expected to be very few alternative forums where the plaintiff has a right to sue. He said the transferee court in this case plainly had and has jurisdiction to adjudicate this action with the defendants acquiescence. Transfer cannot be made under the statue unless it is found to be in the interest of convenience and in the interest of justice. Choice of Law A. Introduction 1. Where does the court look to find the applicable legal rules (to what body of law does the court turn to resolve legal issues which arise in the action 2. Contracts: many contain a choice-of-law provision stipulating the law to be applied in construing and enforcing the contract 3. Courts generally apply its own procedural rules, even in cases where it will apply the substantive rules of another jurisdiction

B. The Erie Doctrine 1. Swift v. Tyson: decided whether the case should be governed by NY contract law, under which the fraud tainting the transaction provided a defense for Tyson (defendant), or by the new law of negotiable instruments that was developing in recent English decisions, under which Tyson would have to pay Swift if Swift had accepted the instrument without notice of the fraud. -Looked to the Rules of Decision Act -The phrase laws of the several states encompassed only statutory law (that is, the Act commanded federal courts to follow the state rule only if it was in a state statute) then the federal court in Swift was free to use the emerging rule or any other it felt was best. -Federal courts exercising jurisdiction on the ground of diversity of citizenship need not, in matters of general jurisprudence apply the law of the state as declared by its highest court; they are free to exercise an independent judgment as to what the common law of the state is or should be. Choice of Law -As between federal courts and state courts -Swift v. Tyson *Rules of Decision Act *Does this apply only when the statue mentions the phrase the laws of several states The New York law was not statutory. Is common law included? *When people spoke of laws they typically meant those enacted by the legislature and long standing local customs. *Judge made law is evidence of what the law is *The Rules of Decision Act would require the court to apply the state statue. Without it, the federal courts were free to apply their own understanding of what the law was. Erie R. Co. v. Tompkins P. 379 Supreme Court of the United States, 1938 304 U.S. 64, 58 S.Ct. 817, 82 L.Ed. 1188 Facts: Harry Tompkins was walking home parallel to the Erie railroad tracks when he was struck by an object protruding from a passing train. Under PA law, a traveler like Tompkins is regarded as a trespasser to whom the railroad merely owes a duty to avoid wanton negligence. Most states rule that the railroad owes a duty of ordinary care. The lawyers tried to avoid harsh PA rule by suing the NY based railroad in federal court. (In Swift, absent statutory law, federal courts apply general law) Procedure: Verdict for Tompkins for $30,000 after the judge applied general law. Affirmed by Court of Appeals. Certiorari granted. Reversed and remanded.

Issue: Is there a such thing as general law in the federal courts and as such, is Swift still good law? Held: No. Analysis: The Court of Appeals said that when courts apply general law, they exercise their independent judgment as to what the law is. The Supreme Court reviewed the holding in Swift and the criticisms of the decision over the years. They said the benefits from the holding were never realized The court said that the law to be applied in ANY case is the law of the state. And whether the law of the state is determined by the Legislature in a statute or by its highest court in a decision is not a matter of federal concern. They said there is no federal general common law. They said that Congress has no power to declare substantive rules of common law applicable in a state, nor does any clause in the Constitution confer such power upon the federal courts. Justice Butler dissented. He said that the doctrine in Swift was not challenged for 50 years. He said the constitutional validity of the rule need not be considered because under PA common law and throughout the country, the evidence required a finding that the plaintiff was guilty of negligence that contributed to cause his injuries. Justice Reed concurred. He agreed Swift should be overruled and courts should follow state law written or unwritten. Erie -Brandeis said Swift was wrong in opinion: -More Recent research of a competent scholar, who examined the original document, which established that the construction given to it by the Court was erroneous; and that the purpose of the section as mererly to make certain that, in all matters except those in which some federal law is controlling, the federal courts exercising jurisdiction in diversity of citizenship cases would apply as their rules of decision the law of the state, unwritten as well as written. (pg. 409) - Says about Swift v. Tyson: doctrine from it made it impossible for equal protection under the law, prevented uniformity in the application of the state law. -Issue: the duty of care owed by a landowner to a trespasser on his land -Lex Loci Delecti: apply law of state of where the wrong occurred - in that era, NY state court would have applied PA state common law. Forum Shopping- shop around to find forum most likely to yield a favorable result for your client professional obligation. -System not obligated to give this. 2. Erie: in diversity cases, federal courts are to apply state substantive law and federal procedural law 3. York: Outcome determinism: in all cases where a federal court is exercising jurisdiction solely because of the diversity of citizenship of the

parties, the outcome of the litigation in the federal court should be substantially the same, so far as legal rules determine the outcome of a litigation, as it would be if tried in a state court. 4. Ragan: Plaintiff filed within two year statue of limitations. Service was not made until statue had run. Rule 3 F.R.C.P: action commenced when complaint filed. Kansas law: service must be made within statue of limitations. Court held Kansas law should apply, action time barred. 5. Woods v. Interstate: Foreign corporation doing business in Mississippi brought action there. It had not registered to do business there. Mississippi had door closing statue barring unregistered foreign corporations from maintaining such actions. Supreme Court held Mississippi statue barred the action. 6. Cohen v. Beneficial: Federal district court had to apply NJ statue even though the federal rule did not impose the same requirement as the NJ law. Hanna v. Plumer P. 400 Supreme Court of the United States, 1965 380 U.S. 460, 85 S.Ct. 1136, 14 L.Ed.2d 8 Facts: On February 6, petitioner, a citizen of Ohio, filed her claim in the District Court for the District of Massachusetts. She claimed damages for personal injuries resulting from a car accident in South Carolina, allegedly caused by the negligence of Louise Plumer Osgood, a Massachusetts citizen deceased at the time of the filing. Respondent, Osgoods executor and a Mass. Citizen, was named defendant. On February 8, service was made by leaving summons and complaint with the respondents wife at his residence in compliance with rule 4(d)(1). On February 26, his answer said the service was invalid under Mass. Law. Mass. Law requires in hand service of process in a situation such as this one. Procedure: Defendant motion for summary judgment granted, holding Mass. Law applied. Appeal. Affirmed holding the conflict was over a substantive matter. Certiorari granted. Reversed. Issue: Whether, in a civil action where the jurisdiction of the United States District Court is based upon diversity of citizenship between the parties, service of process shall be made in the manner prescribed by state law or that set froth in Rule 4(d)(1) of the Federal Rules of Civil Procedure. Held: The adoption of rule 4(d)(1), designed to control service of process in diversity actions, neither exceeded the congressional mandate embodied in the Rules Enabling Act nor transgressed constitutional bounds, and that the Rule is therefore the standard against which the District Court should have measured the adequacy of the service. Analysis: The court said the process of notifying a defendant relates to the practice and procedure of the district courts. They said essentially every

procedural variation is outcome determinative. In this case, applying state versus federal rules would only change how service of process was made. Justice Harland concurred. Notes and Questions on Hanna v. Plumer -Most recent comprehensive statement about what the Erie doctrine means. -Service can be affected by leaving a copy at the defendants abode with someone of reasonable age (federal) *Question 1: Mass: In actions of this kind, it requires delivery in hand -Forum shopping: in favor of the court that will find in the plaintiffs favor (not illegal or immoral) It is an attorneys job to select the most favorable forum. *Question 2: Outcome determinative conflict -By the time the district court decided the case, the statute of limitations for Mass. Would have run. So, after they decided which law would apply, it would no longer matter because they could no re-serve process. -If the executor distributes the assets without assuring that all outstanding creditors have been paid, he may be held personally liable. This is why Mass. has the stricter service of process law -Is the federal court free to follow the federal way of doing it or is it obligated to follow the state rule. Where there is not a F.R.C.P, the question is whether or not the issue is outcome determinative. They also ask if giving the federal courts the right to chose will promote deterring forum-shopping and avoidance of inequitable administration of the laws. -When there is a federal directive on point, court follows that (i.e. FRCP). -Supreme court has never held FRCP invalid. -But when no federal directive on point, question becomes does Federal Judge have to follow State law or can she ignore it? (Erie Question). Black letter: if substantive, need to follow it. But what is substantive? Three Tests: (laid out by Hanna v. Plumer) -Outcome Determinative (York) if effects outcome, then substantive. -Balance the Interests (Byrd) - federal question should usually follow state law, unless there is a federal interest doing it differently. - Twin-aims of Erie -avoid forum shopping. if issue would lead to forum shopping, should follow state law. (Shady Grove) -avoid inequitable administration of the law -

-Question to Ask to determine constitutionality: Whether it is rationally capable of being classified as procedural? Rationally capable of being classified as substantive? If rational, then constitution. *Question 3- How should Woods be decided under Hanna? -There is no F.R.C.P. on foreign corporations registering in states in relation to a close-door statute, and therefore the FRCP does not conflict with the statute. As a result, no conflict = no Hanna . *Question 4- How is Ragan different than Woods? There was a F.R.C.P in Ragan (Rule 3). -Where there is an valid, applicable rule of federal procedure and a conflict with a state rule, they can follow that. How do they determine if it is valid? -Procedural: rationally capable of classification of either substantive and procedural -Ragan: a regulation of procedure. Ragan is no longer good law because it was a procedural rule so it should have been applied under F.R.C.P. Unless, there was no federal rule on point. -Walker: no federal law on point, so state law has to be applied -Where there is an applicable federal rule, it is going to trump state law (Regardless of how much Unequal application and Forum Shopping took place) *Is the rule on point? If so, *Is it a valid rule? -Acts of congress are treated similar to the F.R.C.P. *Is it constitutional? If yes, it must be followed -How does the federal court determine what the applicable state law is? First, you must determine what states law to look to. Traditionally, it was the place of the wrong. -McKenna v. Ortho: diversity personal injury action. Plaintiff alleged she suffered injuries using a prescription drug manufactured by Ortho. She lived in Ohio and suffered all injuries in Ohio. Action was brought in federal district court in PA. Ohio substantive law would apply under PA choice of law rules. They had a conflict of the application statue of limitations (Ohio or PA). They both had a 2 year limitation, but they were different in regards to when the statue started running. (PA used the discovery rule: the cause of action would have been timely). In Ohio, it may have been untimely. *Most states say statue of limitations is procedural. PA has a borrowing action. It will be time barred in PA if it is time barred in the state where the action arose. *Would a PA court applying the borrowing rule look to it with respect to the period or when the action was deemed to accrue. *PA court would look to Ohio law in its totality.

7. Sibbach v. Wilson: plaintiff sued defendant in Illinois federal court for damages inflicted in Indiana. Illinois policy forbid compulsory physical examinations. Federal Rule 35 allowed it. Supreme Court affirmed and demanded plaintiff undergo a physical examination. Walker v. Armco Steel Corp. P. 410 Supreme Court of the United States, 1980 446 U.S. 740, 100 S.Ct. 1978, 64 L.Ed.2d 659 Facts: The petitioner was injured on August 2, 1975 in Oklahoma City, Okla while pounding a Sheffield nail into a cement wall. Respondent was the manufacture of the nail. The petitioner claimed there was a defect in the nail and that it was as a result of the defendants negligence. Petitioner is a resident of Okla. Respondent is a foreign corporation with its principal place of business in another state. The complaint was filed on August 19, 1977. Summons was issued the same day, but service of process was not made until December 1, 1977. The complaint was filed within the 2 year statue of limitations for Okla., but Okla. does not consider the action commenced until service of process is made. But, if the complaint is filed within the time period, they have 60 days to serve process even if that ends up outside the statue of limitations. This service was made outside of the 60 days. Federal Rules of Civil Procedure would have allowed the action. Procedure: District Court dismissed. Appeals court affirmed. Certiorari granted. Affirmed. Issue: Whether in a diversity action in the federal court should follow state law, or alternatively, Rule 3 of the Federal Rule of Civil Procedure in determining when an action is commenced for the purpose of tolling the state statute of limitations. Held: State law. Analysis: In Ragan, the test was whether the Rule was within the scope of the Rules Enabling Act, and if so, within a constitutional grant of power such as the Necessary and Proper Clause of Art. 1. The court first looked to see if the scope of the Federal Rule is sufficiently broad to control the issue. The court said that Rule 3 governs the date from which various timing requirements of the Federal Rules begin to run, but does not affect state statue of limitations. The said that the Okla. Statue is a statement of substantive decision. 8. Klaxon Co. v. Stentor Electric: Supreme Court held that federal district courts must apply the conflicts-of-law rules of the states in which they sit. That means the whole law of the state including choice of law rules. Mason v. American Emery Wheel Works P. 434 United States Court of Appeals, First Circuit, 1957. 241 F.2d 906

Facts: Whit Mason, a citizen of Miss. Filed his complaint in federal court in Rhode Island against The American Emery Wheel Works, a Rhode Island Corporation. Procedure: Defendant motion to dismiss granted. Remanded. Issue: Held: Analysis: Since the injury was inflicted in Miss. The district court deemed itself obliged to apply Miss. Local law to determine the tort liability. Mississippi law would hold that the manufacturer is not liable for negligence where there is no privity of contract between the user and the manufacturer. The court said that a decision may become so illogical that it loses its persuasive or binding force even in the inferior courts. 9. Certification: a procedure that allows the court of one system to petition the court of another system for the answer to an unresolved legal question. Certification Rules -Questions of state law do not only arise in diversity actions. (Most commonly there) -44 states have adopted a certification rule by statute or other -When the federal court certifies, it stays proceedings on the action and certifies a certain question or questions to be answered by the state court. The state will answer with an opinion. The federal court then reactivates the case. *Not all states have them *Their scope varies considerably *It is not mandatory, it is a matter of discretion. -Suppose there is a disputed novel issue of Illinois law in the 3rd circuit. The Illinois decisions give no clear answer. Under Illinois certification, PA cannot certify to them. They also cannot decline to hear it. What do they do? *The federal court simply has to use its best judgment and make a judgment as to what it thinks the highest court of the relevant state would decide. *To make an Erie predication they should evaluate all relevant decisions of applicable state law -The decisions of intermediate appellate courts are not controlling for Erie purposes if the federal courts decide they believe the Supreme Court would decide differently. -If a district court judge disagrees with the ruling of the state supreme court applying another states law, what do they have to do? He has an obligation to decide it the way the PA supreme court would apply it. Otherwise, there would be conflicting applications of law. William Fiore v. Gregory White P. 210 M

Supreme Court of the United States 1999 Facts: William Fiore and David Scarpone were convicted of violating the law forbidding any person to operate a hazardous waste facility without a permit. They both appealed. Fiores conviction was affirmed, Scarpones was reversed. The PA Supreme Court denied review of Fiores case. They reviewed Scarpones and held that the statutory provision did not apply to those who possessed a permit but deviated radically from the permits terms. The PA court again denied Fiores request. They had a permit but operated against its terms. Procedure: Habeas petition granted. Appeals reversed. Certiorari granted. Issue: Whether the Federal Constitution requires that Fiores conviction be set aside in light of Scarpone. Held: Invoked certification procedure to obtain courts view of the matter. Analysis: The court has to determine whether PA considers Scarpone to have explained what the statute always meant, or whether the considered Scarpone to have changed the law. Pleading Pleading 8a Code Procedure/Pleadings -Late 19th early 20th century. Replaced the complexity of common law pleadings. -Some states still follow the code pleading rules. A. Commencing the Action 1. Gillispie v. Goodyear Service Stores: plaintiff required to make plain and concise statement of the facts constituting a cause of action. Hers did not disclose what occurred, when it occurred, where it occurred, who did what, the relationships between defendants and plaintiff or any other factual details. Gillispie v. Goodyear -Does the complaint state facts (if taken as true) sufficient to constitute any cause of action -Mrs. Gillispies complaint did not suffice. She did not plead facts, she pleaded legal conclusions. -Major deficiency was the what and how -I.e. trespass and assault are legal conclusions -The defendants demurrer. They challenge the legal sufficiency of the plaintiffs complaint. They admit to the facts but assert that this does not entitle the plaintiff to recover *Not always called demurrer.

*Challenges facial validity of plaintiffs complaint. *I.e. fails to state a cause of action *In common law, the defendant admits the plaintiffs facts. If the legal theory of demurrer is rejected, judgment was entered for the plaintiff. -Demurrer does NOT admit legal conclusions. It only admits facts. -Why is it so important to have facts? Its necessary to inform the defendant what the plaintiff intends to prove at trial. (factual disclosure of the plaintiffs case so the defendant can properly prepare). It also eliminates unmeritorious cases and eliminates the need for expensive discovery. We want specificity in issues. The court needs to declare the law upon the facts stated. -Judge declaring the law upon the facts stated: ruling on a demurrer. -Summary judgment is another method for weeding out unmeritorious claims. It involves considering information unearthed in discovery and takes place late in the proceedings. -Policy choice -There is no logical distinction between statements that are facts and conclusions Dioguardi v. Durning P. 517 United States Circuit Court of Appeals, Second Circuit, 1944 139 F.2d 774 Facts: Plaintiff brought action against the Collector of Customs at the Port of New York for $5000. Procedure: Complaint dismissed by district court with leave to amend for failure to state a cause of action. Dismissed again. Reversed and remanded. Issue: Did the plaintiffs complaint display a short and plain statement of the claim showing that the pleader is entitled to relief? Held: Yes. Analysis: The court said it believed he stated enough to withstand a mere formal motion directed only to the face of the complaint. The court said there only has to be a short and plain statement of the claim showing that the pleader is entitled to relief. The court believed the plaintiff adequately did this. Dioguardi v. Durning -Under the federal rules, the plaintiff does not have to state facts sufficient to constitute a cause of action. All you need is a complaint with a short and plain statement of the claim stating the type of relief. -He was alleging the existence of a bailment. The collector of customs had custody of Dioguardis property *All you have to prove at trial is that there was a bailment and that you did not get the property back -The Gillispie court would probably have no difficulties with his complaint

2. Conley v. Gibson: The rule is that a complaint should not be dismissed for failure to state a claim unless it appears beyond doubt that the plaintiff can prove no set of facts in support of his claim which would entitle him to relief. Conley v. Gibson -Leading case from the Supreme Court on the subject of pleading requirements in the federal courts. -Rule: a complaint should not be dismissed for failure to state a claim unless it appears beyond doubt that the plaintiff can prove no set of facts in support of his claim which would entitle him to relief. -Appendix of forms. It does not seem the federal rules have a problem with pleading legal conclusions. 3. Leatherman v. Tarrant County Narcotics Intelligence & Coordination Unit: Supreme Court held that a federal court may not apply a more stringent pleading standard in civil rights cases alleging municipal liability under 42 U.S.C. 1983. 4. Bautista v. Los Angeles County: Federal civil rights action. Each individual plaintiff did not set out a claim that he or she is a member of a particular protected class, was qualified and applied for the position he or she sought, and was rejected on prohibited grounds. Bell Atlantic Corporation v. Twombly P. 527 Supreme Court of the United States 2007 U.S., 127 S.Ct. 1955, 167 L.Ed.2d 929 Facts: The plaintiffs were subscribers to local telephone services. They filed a class action suit against the major telephone companies in the U.S. alleging that they had violated section one of the Sherman Antitrust Act. Procedure: District court dismissed the complaint for failure to state a claim upon which relief can be granted. Court of appeals reversed. Certiorari granted. Reversed. Issue: Whether a section one complaint can survive a motion to dismiss when it alleges that major telecommunications providers engaged in certain parallel conduct unfavorable to competition, absent some factual context suggesting agreement, as distinct from identical, independent action. Held: No, it should be dismissed. Analysis: The court was seeking to determine what a plaintiff must plead in order to state a claim under section one of the Sherman Act. Federal Rules of Civil procedure require a short and plain statement of the claim showing that the pleader is entitled to relief, in order to give the defendant fair notice of what the claim is and the grounds upon which it rests. The court said stating such a claim requires a complaint which enough factual matter (taken as true) to suggest that an agreement was made. An allegation of parallel conduct and a bare assertion of conspiracy will not suffice. They must be placed in a

context that raises a suggestion of a preceding agreement, not merely parallel conduct that could just as well be independent action. The court requires only enough facts to state a claim to relief that is plausible on its face. Because the plaintiffs have not nudged their claims across the line from conceivable to plausible, the complaint should be dismissed. Justice Stevens and Ginsburg dissented. They said the complaint described a variety of circumstantial evidence and makes a straightforward allegation. Bell Atlantic Corp. v. Twombly -Massive burdensome discovery that is characteristic of antitrust cases -There must be an actual agreement to engage in anticompetitive conduct to violate the Sherman Act. *The complaint did not allege an agreement (according to the court) -Pleaded too much. Did not allege and essential element of their claim. -Twombly standard: plausibility. Complaint must allege enough facts to show that the claim being asserted by the plaintiffs is a plausible claim. This does not mean the most probable explanation or even possible. -Must look at it in the context of the applicable substantive law Ashcroft v. Iqbal Handout Supreme Court of the United States 2009 129 S.Ct. 1937 Facts: Javaid Iqbal is a citizen of Pakistan and a Muslim. After 9/11 he was arrested and detained by federal officials. He claims he was deprived of constitutional protections while in custody. He filed a complaint against numerous federal officials. The complaint alleges that they adopted an unconstitutional policy that subjected Iqbal to harsh conditions of confinement on account of his race, religion, or national origin. Procedure: Defendant motion to dismiss denied. Court of appeals affirmed. Reversed and remanded. Issue: Did respondent, as the plaintiff in the District Court, plead factual matter that, if taken as true, states a claim that petitioners deprived him of his clearly established constitutional rights. Held: No. The respondents complaint fails to plead sufficient facts to state a claim for purposeful and unlawful discrimination against petitioner. Analysis: The court said that the plaintiff must plead and prove that the defendant acted with discriminatory purpose. Purposeful discrimination requires more than intent as volition or intent as awareness of consequences. To survive a motion to dismiss, the complaint must contain sufficient factual matter, accepted as true, to state a claim to relief that is plausible on its face. A claim has facial plausibility when the plaintiff pleads factual content

that allows the court to draw the reasonable inference that the defendant is liable for the misconduct alleged. Relying on Twombly, the court said the respondents complaint did not nudge his claims of invidious discrimination across the line from conceivable to plausible. Justice Souter dissented. He said the majority misapplied the pleading standard under Bell Atlantic Corp. v. Twombly. He said the complaint satisfied Rule 8a2. He said the complaint contained many allegations linking the defendants to the discriminatory practices of their subordinates. Ashcroft v. Iqbal Two prong approach: o identify allegations in complaint that are not assumed to be true. o Pass the plausibility test. B. Responsive Pleading 1. Cobell v. Norton: Under Rule 12(f), immaterial matter consists of statements and averments being no essential or important relationship to the claim for relief or the defenses being pled. Responsive Pleading 8b -Defendants responding to factual allegations of plaintiffs complaint Defensive pleadings -How defendant must respond to Plaintiffs complaint. Look at rule 12 requires that the defendant respond to a complaint within 20 days of service or a default judgment will be entered. -Motion to strike: rarely granted. Rigorous burden to meet. Usually has to be prejudicial to the opposing party. o *Legally insufficient defense. Becomes the plaintiffs equivalent of a demurrer. Zielinski v. Philadelphia Piers, Inc. P. 569 (one of Cannons favs) United States District Court, Eastern District of Pennsylvania, 1956. 139 F.Supp. 408 Facts: The plaintiff was working on Pier 96 in Philadelphia. He received injuries while operating a fork lift and collided with another fork lift. Procedure: Issue: Held: Analysis: PA courts hold that an allegation of agency in the complaint requires a statement to the jury that agency is admitted where an attempt to amend the answer is made after the expiration of the period of limitations.

Zielinski -Respondeat superior assertion. Operator of fork lift was agent, servant of PPI in the course of his duty as such. -PPI had leased its equipment to another company, CCI, and Johnson had ceased to be a PPI employee and had become an employee of CCI. Therefore, PPI would not be liable for Zielinskis injuries -PPIs denial was an ineffective denial of the part of paragraph 5 that made the agency allegation. Should file a more specific denial than a general denial. o *General denial: 8b3, deny all of the allegations of a pleading including assertion of the grounds for the courts jurisdiction. Very difficult to use. o *8b4,2: if you dont intend to deny every part, must specify which parts are being admitted and which parts are being denied. There was no dispute about the accident, and they did not intend to deny everything in paragraph five. They should have specified. o *If you deny improperly you are deemed to have admitted -PPI should be estopped. Need some sort of misrepresentation, and other side need to have relied on the misrep. -CCI and PPI both insured by same carrier. Case from insurers perspective: drag PPI case as long as possible before they are found not liable, and then by the time case is filed against CCI, statute of limitations would run out. -Improper denial is ineffective denial and thus an admission. Most common to occur with lack of knowledge or information sufficient to form a belief about the truth of an allegation. -Admit generally or deny specifically or vise versa 2. Motion to strike: the mechanism for challenging the substantive sufficiency of defenses raised in an adversarys answer or other responsive pleading 3. Motion for judgment on the pleadings: a method of attacking the substantive sufficiency of an opposing partys pleading after all the pleadings have been completed. *It looks to the face of all the pleadings rather than a single pleading. It asserts that on the basis of all the pleadings, the moving party is entitled to judgment as a matter of law. 4. Improper Forms of Denial -Denials for lack of information: lacks knowledge or information sufficient to form a belief about the truth -Negative pregnant denials: an admission that defendant owes plaintiff one cent less than specified -Conjunctive denials: evasive and admit certain facts 5. Affirmative Defenses

-Ones that admit the allegations of the complaint but suggest some other reason why there is no right of recovery -Ones that concern allegations outside of the plaintiffs prima facie case that the defendant therefore cannot raise by a simple denial in the answer Ingraham v. United States P. 574 United States Court of Appeals, Fifth Circuit, 1987 808 F.2d 1075 Facts: The plaintiffs sued the United States for severe injuries caused by the negligence of government physicians. Procedure: Judgment for plaintiffs. Government moved for relief from the judgment. Denied by district courts. Affirmed. Issue: Was the defense an affirmative defense? Held: Yes. The government did not raise the issue timely before the trial courts and the issues were not preserved for appeal. Analysis: The court looked to determine if the defense was an affirmative defense, because there are different rules of timeliness for affirmative defenses. Affirmative Defenses -Rule 8C. If not affirmatively stated by defendant, it is waived. -Ingraham: prevention of unfair surprise (do not want the plaintiff to be ambushed by unanticipated defense at trial) o *Statutory cap on damages was an affirmative defense. o *Intrinsic element of plaintiff cause of action versus extrinsic (not part of plaintiffs case. If its going to be in the case defendant has to assert it). o *Confession and avoidance: defendant admits to the claims and that they entitled the plaintiff to recover, but there are some extrinsic facts outside the complaint that avoids the normal legal effect that bars him from recovering. -Burden of persuasion: who has the ultimate burden of persuading the trier of fact that his version of the facts is the correct version (only comes in to play at the end of the trial) -Burden of production: who has the burden of producing evidence at trial, sufficient to take the issue to the jury -Burden of pleading: who has the burden of pleading. Ordinarily, we assign the burden of pleading to the party who has the burden of production. How do we decide how to assign the burden of production? Ease of access to the evidence, other policy considerations. -Characteristics in common with affirmative defenses: unusual/novel (such as caps on damages) o *If in doubt, plead it as an affirmative defense. If you dont plead it, you may have been found to have waived it.

-Motion for a more definite statement o *Very rarely granted o *United Aircraft: defendant moved for a more definite statement under 12(e) demanding the plaintiff identify by name the strikers involved and specify the jobs for which they were qualified. ~What was the plaintiffs problem? The alleged a violation of a strike settlement contract in that the defendant-employer had failed to recall strikers to work as jobs for which they were qualified become available. ~How did the court resolve it? They granted the motion for a more definite statement but deferred the ruling. They said the union would not have to provide the information until it had completed discovery. ~What standard must a party meet in order to be entitled to a more definite statement? Only used to attack a pleading to which you have to respond to. Have to persuade that it is so vague and ambiguous that they cannot frame an answer. ~How else could United have responded? Rule 8e5 -Rule 12c: Motion for judgment on the pleadings: asserts on the basis of uncontradicted facts the mover is entitled to judgment as a matter of law. There are no undisputed factual issues to resolve. If there are no factual disputes, there is no need for a trial. o *Look at all of the pleadings.

THE PROBLEM OF TRUTHFULNESS IN PLEADING Rule 11 Rule 11: applicable only to civil actions in the district courts. No application to costs incurred at the appellate level. Pleadings in the federal courts do not have to be verified The American Rule: every party to litigation bears his own costs. The winner is usually not entitled to recover his attorneys fees. There are exceptions. Before triggering the certification provision, the lawyer must present paper (pleading, written motion, or other paper) to the court 1983 rule: the lawyer had to sign the paper to effect certification The 1993 rule made it more rigorous 11(b3,4): factual matters. As long as what the client says is objectively reasonable, the lawyer is entitled to rely upon that information. 11b2: Legal contentions. Addition of the word nonfrivolous (obviously without merit) 11c: Sanctions: it is under the courts discretion whether they are going to give sanctions

o *Also section 1963: requires a showing of subjective bad faith on the part of the attorney involved Amendments and Supplemental Pleadings Beeck v. Aquaslide N Dive Corp. P. 579 United States Court of Appeals, Eighth Circuit, 1977 562 F.2d 537 Facts: Jerry Beeck was severely injured while using a water slide. He and his wife sued Aquaslide alleging that it manufactured the water slide. Aquaslide initially admitted manufacturing the slide, but later moved to amend its answer to deny manufacturing it. Aquaslides insurer believed it was manufacturer by Aquaslide. The president of the company inspected it and said it was not their product. Procedure: Defendant motion to amend answer resisted by plaintiffs. District court granted leave to amend. Verdict for defendant on issue of whether they designed, manufactured, or sold the slide. Summary judgment to dismiss the case. Appeal. Issue: Where the manufacturer of the product admitted in its answer and later in its answer to interrogatories both filed prior to the running of the statute of limitations that it designed, manufactured and sold the water slide in question, was it an abuse of the trial courts discretion to grant leave to amend to the manufacturer in order to deny these admissions after the running of the statue of limitations? Held: They did not abuse their discretion. Analysis: In ruling on a motion for leave to amend, the trial court must inquire into the issue of prejudice to the opposing party, in light of the facts of the case. The allowance or denial of leave to amend may only be reviewed for an abuse of discretion. The court said the district court clearly searched the record for evidence of bad faith, prejudice and undue delay which might be sufficient to overbalance the mandate of Rule 15a. A. Moore v. Moore: The court concluded that the defendant was on timely notice that the court would decide not merely whether he was entitled to custody but would determine who was entitled to custody. He can claim no surprise or lack of notice or opportunity. The trial court abused its discretion in permitting amendment of the pleadings to include separate maintenance and in making such an award. The defendant impliedly consented to try the issue. The attorney failed to object to the evidence as being outside of the issues contained in the pleading. Worthington v. Wilson P. 585 United States District Court, Central District of Illinois, 1992 790 F. Supp. 829 Facts: Richard Worthington was arrested by two police officers. At the time, he had an injured hand. The officers arresting him handled him roughly and

worsened the injury. He claimed the officers violated his constitutional rights in violation of the Civil Rights Act of 1964. Worthington filed an amended complaint. Defendants moved to dismiss on the grounds that the statute of limitations had run and the complaint failed to state a proper claim. Procedure: Filed in Circuit Court of Peoria County. Removed to US District Court. Defendant motion to dismiss and motion for sanctions allowed. Issue: Held: Analysis: The statute of limitations is two years. The plaintiffs action must have been filed by February 25, 1991. The amended complaint was not filed until June 17, 1991. Therefore, the only way the complaint can be found to be timely is if it relates back to the filing of the original complaint. Likewise, the new party must have received notice of the action before the statue of limitations expired. The defendants did not receive notice until February 25, 1991. Under the old rule, the amended complaint would not relate back and would be untimely filed. Under the new rule, an amended complaint which changes the name of a defendant will relate back to the filing of the original complaint if it arises out of the same conduct contained in the original complaint and the new party was aware of the action within 120 days of filing of the original complaint. Under the new rule, the complaint is timely because the defendants received notice of the action within 120 days of the original filing. However, the seventh circuit says that an amended complaint which replaces fictitious names with actual names due to an initial lack of knowledge concerning the proper defendant does not involve a mistake and is therefore not entitled to relation back under rule 15a. Therefore, Worthingtons amended complaint is not entitled to relation back. The court did not need to address the effect of state law because they said it was inapplicable in this action. The federal courts absorb state law only when federal law neglects the topic, which is not the case here. B. Supplemental pleading: can be used to cure defects in the original pleading, to add new claims, or the provide additional facts that update the complaint. AMENDMENTS TO PLEADINGS Rule 15 -Govern amendments to pleadings in the federal courts -Problem 1: Did Aquaslide have a right to file an amended answer at this point? Yes, they filed the amendment within 20 days after serving the pleading and any party may amend its answer once. No responsive pleading to an answer is allowed. o It does not have to ask the court if done within the 20 day period.

o Have to determine if the first day is included, because if it is, then it would not have been timely. Rule 6a says the first day is not included. -Problem 2: Can amend once before being served with a responsive pleading. Is 12b6 motion for failure to state a claim a responsive pleading. If it is, then no amendment can be made. It is not a pleading because rule 7a says it is not. -Problem 3: The answer is a responsive pleading. The plaintiff would request leave to amend by filing a motion with the court. The burden is to persuade the court that it is in the interest of justice. o Beeck v. Aquaslide: If the proposed amendment would be sufficient to survive a 12b motion, it should be granted. The burden is on the party opposing the amendment. The rule is biased in favoring amendments. o Prejudice must be shown to defeat a motion to amend. Prejudice arising from delay (such as the running of the statute of limitations, evidence being lost, witnesses dying) o When Aquaslide admitted, they acted in good faith and exercised due diligence. They relied on reports from three separate insurance companies. o Prejudice is not always sufficient. There may be a requirement of more. -Problem 4: Are these legally valid defenses? Rule 12h1b. As a matter of course, assert threshold defenses in a preliminary 12b motion, in a responsive pleading, or in an amendment to the answer as a matter of course. If you have to ask the court it is not a matter course. Not a legally sufficient defense. o -Moore v. Moore: there was no implied consent because the evidence was not so uniquely pertinent to her support alone. Lawyer failed to object to relevance of those needs. He could have amended the pleading. -Question 5: The statute of limitations has run. If he had filed a new action instead of amending the complaint, that action would be time barred. Rule 15c: does it relate back to the date of the original pleading? If it relates back, it bars the statute of limitations. He is attempting to assert a new claim against the same defendant. No relation back if there is a distinct and separate cause of action (Some states) Question 6: Same claim for the same injury, just be asserted against a separate defendant. The new defendant has to have received notice of the actions. They received notice within 120 days. The claim against CCI would not be barred by the statute of limitations. But, PPI received notice, CCI may not have (although they contested the suit so they must have received notice) -Question 8: Argue the amendment would not relate back. A lack of knowledge is not the equivalent of a mistake (i.e. the use of fictions

names for defendants who are known to exist but their exact identities are not known. Do we know if Horney received notice? (We can infer from the fact that it was widely publicized in NFL circuits that he did). Krupski v. Crociere had knowledge and didnt understand it, and therefore names wrong party, therefore leaves it open as a mistake. DISCOVERY A. Lindberger v. General Motors: Defendants refused to answer interrogatories about changes made to the front loader since the accident. Court said the information sought is relevant to the subject-matter of the action. It may also be relevant on the issues of negligence and contributory negligence B. World Wrestling Federation Entertainment, Inc. v. William Morris Agency, Inc: the court refused to allow plaintiff to discovery defendants contractual agreements with third parties. Hickman v. Taylor P. 831 Supreme Court of the United States, 1947 329 U.S. 495, 67 S.Ct. 385, 91 L.Ed. 451 Facts: On February 7, 1943, the tug J.M. Taylor sank while helping tow a car float owned by the Baltimore & Ohio Railroad across the Delaware River in Philadelphia. The cause of the accident was unknown. Five of the nine crew members died. The tug owners answered all of the interrogatories except number 38. They admitted that statements of the survivors had been taken, but they declined to summarize or set forth contents. Procedure: District court held the matters were not privileged, ordered them to answer the interrogatories. Court of Appeals reversed, held that the information was the work product of the lawyer. Certiorari granted. Affirmed. Issue: What is the extent to which a party may inquire into oral and written statements of witnesses, or other information, secured by an adverse partys counsel in the course of preparation for possible litigation after a claim has arisen. Held: Analysis: Rule 33 did not permit the petitioner to obtain such memoranda and statements as adjuncts to the interrogatories addressed to the individual tug owners. The court said the discovery provisions are to be applied as broadly and liberally as possible, and the privilege limitation must be restricted to its narrowest bounds. The court said nothing can justify unwarranted inquiries into the files and the mental impressions of an attorney. The court said when relevant and non-privileged facts remain hidden in an attorneys file and where production of those facts is essential to the preparation of ones case, discovery may be held. It said the petitioner did not show necessity under the circumstances of this case so as to justify production.

Justice Jackson concurred. He said there was no way a lawyer could reproduce the oral statements from a deposition and even if his recollection were perfect, the statement would be his language permeated with his inferences. C. Snead v. American Export-Isbrandtsen Lines, Inc.: The only time there will be a substantial need to know about surveillance pictures will be in those instances where there would be a major discrepancy between the testimony the plaintiff will give and that which the films would seem to portray. DISCOVERY Rule 26 Mandatory Disclosure o Rule 26a1: Each party is obligated up front to disclose certain kinds of information without request. (Any participant was invariably entitled to the information, so why not give it up front.) *Must disclose the name of each individual likely to have discoverable information that the lawyer may use at trial to support claims or defenses (i.e. two favorable witnesses, not the unfavorable witness) 26A2- disclosure of expert witnesses 26A3 disclosures pre-trial disclosure of witnesses and evidence that you intend to offer at trial. o Rule 26(b) Basic question: What is the scope of information that is discoverable? Question of discoverability (much broader) is different than the rules of admissibility at trial. Privilege: incorporates traditional evidentiary principle of privileged. If it would be objected to at trial. Only significant exception. Limitations on the Scope of Discovery - Rule 26b3 Work product: Trail Preparation Materials: Prepared in anticipation of litigation or for trial (Hickman v. Taylor, pg 897) *If prepared in anticipation for trial, it is work product. If it is work product, it is presumptively immune from litigation. *What about in mixed motives cases? Could say if it is merely tainted with the idea of future litigation, it is work product. Or, you should say it has to be very clear. ~Was it prepared in anticipation of litigation? (litigation does not have to be on going when the document was prepared)

~Use in litigation does not have to be the sole purpose -Who prepared or obtained the document? (i.e. a lawyer) When was it made in relation to the underlying evens giving rise to the litigation? What are the possible purposes? What is the degree of routineness of preparing the document? Most important, what is the probability of litigation? (The higher degree of probability, the more likely a court will say it will be prepared for litigation) Must identify experts, and provide written report from expert as to what he will testify at trial (only for those that will testify at trial) There is a distinction between trial experts and those that will not testify at trial

Discovery Mechanisms Rule 30 -Depositions: Rule 30 (big one)/31. Can depose anyone found in the U.S. Rule 33- Written interrogatories: Rule 33 only be served upon parties to the action. Anything that is discoverable from a party in a deposition is discoverable through a written interrogatory. (Probably going to be of little use because it will come from the persons lawyer) o They are effective for hard, objective, underlying, historical facts. o Limited to 25 interrogatories per party. If want to obtain more, need to obtain permission from court. Rule 34- Requests for Production, Rule 34. Tangible evidence of electronic storage. Rule 35- Physical and mental examinations of litigants. Rule 35.Good cause must be shown. o Must file a motion with the court first (unique in this way) or can be done by agreement o Rule aimed at personal injury plaintiff Appellate Review -Generally not appealable because it is not a final order -Reviewable at the end of the case, but generally moot at that point Adjudication Without Trial A. Summary Judgment Lundeen v. Cordner P. 891 United States Court of Appeals, Eighth Circuit, 1966 354 F.2d 401

Facts: Originally, Joseph Cordners children were his beneficiaries. When he remarried, he attempted to change it to his new wife, but he died before the change was completed. The children are arguing that they are still the beneficiaries. There is no dispute that under the documents, the children are the beneficiaries. The dispute is whether the new wife should be because he had tried to change it. The new wife presented a wide range of evidence showing the deceased had tried to change the policy. Procedure: Defendant (wife) move for summary judgment. Granted. Issue: Was summary judgment properly granted? Held: Yes. Analysis: The court said as a general rule of law, an insureds attempt to change his beneficiary will be given effect if all that remains to be done is a ministerial duty on the part of the insurer. If the deceased completed all of the necessary steps required of him to change the beneficiary, the wife would be entitled to judgment. The court said it was clear the deceased actually made a change in the beneficiaries of his life insurance policy. The wording was also shown beyond any reasonable and genuine dispute. 1. Adickes v. S.H. Kress & Co.: Where the evidentiary matter in support of the motion does not establish the absence of a genuine issue, summary judgment must be denied even if no opposing evidentiary matter is presented. Celotex Corp. v. Catrett P. 901 Supreme Court of the United States, 1986 477 U.S. 317, 106 S.Ct. 2548, 91 L.Ed. 265 Facts: The plaintiff filed suit against fifteen companies alleging that her husband died as a result of his exposure to asbestos in their products. As a response to the defendants summary judgment motion, the plaintiff produced documents demonstrating that he had been exposed and there was a material factual dispute. Procedure: Defendant motion for summary judgment granted by District Court. Court of Appeals Reversed. Certiorari granted. Reversed. Issue: Was the Court of Appeals reversal of summary judgment appropriate? Held: No. Analysis: The court said Rule 56(c) does not have a requirement that the moving party support its motion with affidavits or other similar materials negating the opponents claim. A summary judgment motion may be properly made in reliance solely on the pleadings, dispositions, answers to interrogatories, and admissions on file. Justice Brennan dissented. He said the burden of establishing a nonexistent genuine issue has two components: an initial burden of production, which shifts to the nonmoving party if satisfied by the moving party; and an ultimate burden of persuasion, which always remains on the moving party. A party may meet the burden of production by submitting

affirmative evidence that negates an essential element of the nonmoving partys claim, or they may demonstrate that the nonmoving partys evidence is insufficient to establish an essential element of the nonmoving partys claim. He said Celotex failed to meet the burden of production. Summary Judgment -Most are made in favor or defendants -Rule 56 -Usually defendants making a motion for summary judgment -Most state procedures mimic the federal rules -Question 1: Yes. There must be no genuine issue as to any material fact. Can support a motion for summary judgment with pleadings, discovery and disclosure materials on file, any affidavits. He did not respond to the motion. -Question 2: No, there is evidence on both sides. There is a genuine issue as to a material fact. -Question 3: There is an issue a fact if there is evidence on both sides. Ps affidavit meets the requirements. Nothing in the rule that says party cant use his own affidavit. Difference between Celotex and Cross: defendants moving for summary judgment in Celotex, plaintiff in Cross, If evidence is impossible, and fact-finder would have to suspend laws of physics to credit evidence, certainly will not do it.

You might also like